Вы находитесь на странице: 1из 149

МИНИСТЕРСТВО ОБРАЗОВАНИЯ И НАУКИ КЫРГЫЗСКОЙ РЕСПУБЛИКИ

КЫРГЫЗСКИЙ НАЦИОНАЛЬНЫЙ УНИВЕРСИТЕТ


имени Ж. БАЛАСАГЫНА
ПРОФЕССИОНАЛЬНЫЙ КОЛЛЕДЖ ОТДЕЛЕНИЕ ПЕРЕВОДЧЕСКОЕ ДЕЛО

«СОГЛАСОВАНО» «УТВЕРЖДАЮ»
Директор Профессионального
Руководители колледжа
общеобразовательных дисциплин Каныбек у А._________________
(Ф.И.О.)
___________________________________ _______________________________
(подпись) (подпись)
« ____ » ____________ 202__г. « ____ » ____________ 202__г.

УЧЕБНО-МЕТОДИЧЕСКИЙ КОМПЛЕКС

ДИСЦИПЛИНЫ

«Профессиональная математика

Специальность: «Переводческое дело» , «Туризм»

курс 1 на базе 11 класса, курс 2 на базе 9 класса


семестр 1;3
лекции 16 (час.)
практическая работа 16 (час.)
всего часов нагрузки 60 (час.)
самостоятельная работа 28 (час.)
курсовой проект
экзамен 1; 3 семестр

Учебно-методический комплекс составлен в соответствии с требованиями государственного


образовательного стандарта среднего профессионального образования

Учебно-методический комплекс дисциплины обсужден на заседании


_____________________________________________________________________________

Учебно - методический комплекс составила преподаватель: Джолумбекова М.Т.

Аннотация

Настоящий учебно-методический комплекс (УМК) разработан в соответствии


Государственным образовательным стандартом и рабочей учебной программой дисциплины.
Учебно-методический комплекс представляет собой комплект разнообразных
нормативных, учебно-методических, информационных и контролирующих материалов по
дисциплине.
УМК создается для повышения эффективности самостоятельной работы студентов,
качества подготовки специалистов в системе университетского образования, активного
использования в учебном процессе современных педагогических технологий.
УМК вводится в учебный процесс для решения следующих задач:
- освоение студентом в режиме самостоятельной работы дисциплины при участии
преподавателя в качестве консультанта;
- систематизация учебной работы студента в течение семестров; развитие мотивации
обучения у студента; привитие студенту навыков совершенствования и самообразования;
- вовлечение студента в качестве активного участника в открытую креативную
образовательную среду; адаптация студента к условиям деятельности в информационном
обществе.
Учебно-методический комплекс включает в себя:
 аннотацию;
 рабочую программу дисциплины;
 силлабус
 конспекты лекций;
 материалы для практических занятий
 материалы для организации самостоятельной работы студентов
 контрольно-измерительные материалы;
 список литературы;
 глоссарий.
МИНИСТЕРСТВО ОБРАЗОВАНИЯ И НАУКИ КЫРГЫЗСКОЙ РЕСПУБЛИКИ
КЫРГЫЗСКИЙ НАЦИОНАЛЬНЫЙ УНИВЕРСИТЕТ
имени Ж. БАЛАСАГЫНА
ПРОФЕССИОНАЛЬНЫЙ КОЛЛЕДЖ ОТДЕЛЕНИЕ ПЕРЕВОДЧЕСКОЕ ДЕЛО

РАБОЧАЯ ПРОГРАММА
дисциплины

«Профессиональная математика»

для студентов направления Переводческое дело, Туризм


Курс 1 на базе11 класса, курс 2 на базе 9 класса
Семестр 1; 3
Экзамен 1; 3
Всего часов по учебному плану 60
Из них:
 Лекции 16
 Лабораторные
 Практические 16
 Курсовой проект
 Индивидуальные занятия
 Самостоятельная работа 28

Рабочая программа составлена на основании ГОС СПО КР, утвержденного МОиН КР


приказом № 567/1 от «15» мая 2019г. и типового учебного плана по данной специальности,
утвержденному приказом МОиН КР № 443 от «13» апреля 2018 г.

Рабочая программа разработана преподавателем Джолумбекова М.Т.


Одобрено заседании предметно-цикловой комиссии __________________________

Протокол №_____ от «________» ___________20__г.____________________________


(Ф.И.О. председатель ПЦК, подпись.)

Министерство образования и науки Кыргызской Республики


КЫРГЫЗСКИЙ НАЦИОНАЛЬНЫЙ УНИВЕРСИТЕТ
ИМ. Ж.БАЛАСАГЫНА
Профессиональный колледж

«СОГЛАСОВАНО» «УТВЕРЖДАЮ»

Руководители Директор Профессионального


общеобразовательных дисциплин Колледжа Каныбек у А

___________________________________ ___________________________
(подпись)
(подпись)
« ____ » ____________ 202__г.
« ____ » ____________ 202__г.

РАБОЧАЯ ПРОГРАММА

по дисциплине: ___Профессиональная математика_

Специальность подготовки : Переводческое дело, Туризм

050720 -Переводческое дело


100201 -Туризм

Рабочая программа составлена на основании ГОС СПО КР, утвержденного МОиН КР приказом №
567/1 от «15» мая 2019 г. и типового учебного плана по данной специальности, утвержденному
приказом МОиН КР № 443 от «13» апреля 2018 г.

Разработчик рабочей программы: Джолумбекова М.Т. ____________


(должность, Ф.И.О., подпись)
Одобренона заседании предметно-цикловой комиссии

Протокол №_____ от «______»______________20__г. ___________________________


(Ф.И.О. председателя , УМК,подпись)

Курс - 1 на базе 11 класса, 2 на базе 9 класса.

Семестр - 1; 3
Количество учебных недель в семестре - 16

Форма итогового контроля - экзамен

Число кредитов - 2

Всего часов по учебному плану - 60

Всего часов по Количество академических часов


учебному плану Очная
Курс, семестр лек пр (сем) лаб срс

1 курс на 1 60 16 16 28
базе 11 кл семестр
2 курс на 3 60 16 16 28
базе 9 кл семестр

Раздел 1. Общие положения

1.1. Аннотация дисциплины:


Трудно переоценить значение профессиональной математики в системе знаний
образованного человека. Методы, понятия математики позволяют моделировать практически
все наблюдаемые, исследуемые явления в жизни человека. Строгость построения
математической теории является классическим примером построения логически
совершенной теории. Изучение математической теории дисциплинирует саму систему
мышления человека, прививает навыки самостоятельного логического мышления.
Совершенствование методов управления во многом связано с применением в любой науке и
практике разнообразных математических методов исследований существенно расширяет
возможность успешного применения математики. Математические модели в экономике и
интенсивное применение вычислительной техники, которая быстро развивается в последние
годы, расширяет круг применения в различных отраслях экономических науках.

1.2. Компетенции, формируемые в результате освоения дисциплины.


 общенаучными (ОК):
 владеть целостной системой научных знаний об окружающем мире, способен ориентироваться в
ценностях жизни, культуры (ОК-1);
 способен использовать базовые положения математических /естественных/ гуманитарных/
экономических наук при решении профессиональных задач (ОК-2);
 способен приобретать новые знания, с большой степенью самостоятельности, с использованием
современных образовательных и информационных технологий (ОК-3);
 способен анализировать и оценивать социально-экономические и культурные последствия новых
явлений в науке, технике и технологии, профессиональной сфере (ОК-4);
 способен на научной основе оценивать свой труд, оценивать, с большой степенью
самостоятельности, результаты своей деятельности (ОК-5).

1.3. Цель преподавания дисциплины:


 дать студентам математические знания необходимые для изучения специальных, инженерных
дисциплин и развития навыков, требуемых для применения математических методов в
практике инженера.
 формирование математической культуры студентов,
 использования в других областях математического знания и дисциплинах естественно
научного содержания.
 обучение основным математическим понятиям и методам, необходимым для анализа и
моделирования экономических проблем при поиске рациональных решений в сложных
условиях;
 ознакомление студентов с современным математическим аппаратом, необходимым для
анализа вероятностных моделей при решении теоретических и практических задач
экономики;
 развитие навыков самостоятельного изучения учебной и научной литературы по математике
и ее приложениям к экономике и смежным дисциплинам;
 развитие аналитических способностей, необходимых для решения научных и практических
проблем;
 формирование личности студентов, развитие их интеллекта и способностей к логическому и
алгоритмическому мышлению.

В результате изучения дисциплины следует:


- добиться четкого понимания понятий, знания определений и основных
методов профессиональной математики;
- научить решать типовые математические задачи по всем разделам
Задачи учебной дисциплины:
 - ознакомить студентов с основными математическими понятиями и методами;
 - научить грамотной математической речи;
 - познакомить с приемами работы с научной, методической, справочной литературой;
 - привить навыки решения основных типов задач по разделам дисциплины;
 - провести преемственную связь данной дисциплины с физикой, информатикой;
 - подготовить студентов к самостоятельному изучению тех разделов математики, которые
применяются в практической и исследовательской работе специалистов

В результате освоения дисциплины студент должен:


1) Знать
 основные понятия линейной алгебры (матрицы, определители, решение систем линейных
уравнений и неравенств);
 основные понятия и задачи аналитической геометрии (прямая на плоскости, взаимное
расположение прямых на плоскости, кривые второго порядка);
 основные понятия и методы дифференциального исчисления (предел, производная, и
дифференциал функции);
 основные понятия и методы и интегрального исчисления ( первообразная,
неопределенный интеграл, определенный интеграл);
2) Уметь:
 применять математические методы при решении профессиональных задач;.
 решать типовые задачи;
 использовать математический язык и математическую символику при построении
организационно-управленческих моделей;
 дифференцировать и интегрировать;
 использовать пакеты математических программ для решения алгебраических уравнений;
 устанавливать границы применимости методов;
 уметь проверять и анализировать полученные решения.

3) Владеть:
 навыками применения современного математического инструментария для решения
профессиональных задач;
 опытом применения математической символики для выражения количественных и
качественных отношений объектов;
 опытом исследования аналитического решения задач;
 опытом построения математических моделей профессиональных задач и интерпретации
полученных результатов.

Дисциплина «Математика» является базовой дисциплиной естественно- научного цикла

1.4. Задачи преподавания дисциплины сводятся к:

В результате освоения дисциплины обучающийся должен уметь:


 Выполнять арифметические действия над числами, сочетая устные и письменные приемы;
 использовать понятие функции для описания и анализа зависимостей величин;
 находить производные элементарных функций;
 использовать производную для изучения свойств функций и построения графиков;
 применять производную для проведения приближенных вычислений, решать задачи
прикладного характера на нахождение наибольшего и наименьшего значения;
 вычислять в простейших случаях площади и объемы с использованием определенного
интеграла;
 составлять и решать уравнения и неравенства, связывающие неизвестные величины в
текстовых (в том числе прикладных) задачах.
 для построения и исследования простейших математических моделей.
 решать простейшие комбинаторные задачи методом перебора, а также с использованием
известных формул;
 проводить доказательные рассуждения в ходе решения задач;
В результате освоения дисциплины обучающийся должен знать:
-значение математической науки для решения задач, возникающих в теории и практике;
широту и в то же время ограниченность применения математических методов к анализу и
исследованию процессов и явлений в природе и обществе; значение практики и вопросов,
возникающих в самой математике для формирования и развития математической науки;
историю развития понятия числа, создания математического анализа, возникновения и
развития геометрии; универсальный характер законов логики математических рассуждений,
их применимость во всех областях человеческой деятельности; вероятностный характер
различных процессов окружающего мира.

1.5 Взаимосвязь учебных дисциплин:


Пререквизиты :
Дисциплина является продолжением и углубления изучения математики, начатого в
школе. Знание профессионального математики является важнейшей основой при изучении
таких дисциплин, как линейная алгебра, прикладная математика, исследование операций, теория
оптимизации, теория вероятности и математическая статистика. Изучение дисциплины
предусматривает проведение лекционных, практических занятий, самостоятельную работу
студентов.
Постреквизиты: линейная алгебра, прикладная математика

Раздел 2. Содержание дисциплины и формируемые компетенции


Наименование тем и разделов по
Вид контроля Компетенции
№ дисциплине
1 Элементы линейной алгебры. Матрицы. Индивидуальное ОК1-ОК8
Действия над матрицами и векторами . задание
2 Определитель матрицы. Свойства Устный опрос
определителей и их вычисления. Обратная
матрица. Обращение матриц второго и третьего
порядков.
3 Элементы линейной алгебры. Системы Индивидуальное ОК1-ОК8
линейных уравнений. Решение системы задание
линейных уравнений в матричной форме. Устный опрос
4 Элементы линейной алгебры. Решение Решение задач ОК1-ОК8
линейных уравнений по формулам Крамера .
Решение линейных уравнений методом Гаусса.
5 Правила дифференцирования суммы, Решение задач ОК1-ОК8
произведения и частного. Правило
дифференцирования сложной функции.

6 Действительные и комплексные числа. Индивидуальное ОК1-ОК8


Множества. Числовые множества. Множества задание
действительных чисел. Комплексные числа. Устный опрос
Действия над комплексными числами в
алгебраической форме.
7 Неопределенный интеграл. Основные Решение задач ОК1-ОК8
свойства неопределенного интеграла.
Интегрирование подстановкой и по частям.
Определенный интеграл. Вычисление
определенных интегралов. Вычисление
площадей плоских фигур.
8 Элементы комбинаторики и теория Решение задач ОК1-ОК8
вероятностей.
Перестановка.Сочетание.Размешение.

Содержание дисциплины и вырабатываемые компетенции приводятся в виде табл. 2.1.


Количество академических часов
№ Название тем и их содержание Очная
Лек. Практ. СРС

1 модуль
1 Элементы линейной алгебры. Матрицы.
Действия над матрицами и векторами .
2 2 4

2 Определитель матрицы. Свойства


определителей и их вычисления. Обращение 2 2 2
матриц второго и третьего порядков.
Элементы линейной алгебры. Системы
3 линейных уравнений. Решение системы 2 2 2
линейных уравнений в матричной форме.
Элементы линейной алгебры. Решение
линейных уравнений по формулам Крамера .
4 2 2 4
Решение линейных уравнений методом
Гаусса.
Правила дифференцирования суммы,
5 произведения и частного. Правило 2 2 4
дифференцирования сложной функции.
2 модуль
Действительные и комплексные числа.
Множества. Числовые множества.
6 Комплексные числа. Действия над 2 2 4
комплексными числами в алгебраической
форме.
Неопределенный интеграл. Основные
свойства неопределенного интеграла.
Интегрирование подстановкой и по частям.
7 2 2 4
Определенный интеграл. Вычисление
определенных интегралов. Вычисление
площадей плоских фигур.
Элементы комбинаторики и теория
8 вероятностей. 2 2 4
Перестановка.Сочетание.Размешение
Общий объем учебной нагрузки (в часах) 16 16 28
Всего часов:60 32 28

2.2. Практические (семинарские) занятия.


№ Наименование и Вид контроля Знать, уметь ,владеть
краткое содержание компетенции
Занятия.1-модуль
1. Матрицы. Действия над Индивидуальное Знать общие матем. знания,
матрицами. задание термины, символы, выполнение
Определитель матрицы. Устный опрос домашних работ по теме
Свойства определителей
и их вычисления.
Обращение матриц
второго и третьего
порядка
2. Системы линейных Индивидуальное Знать основные логические
уравнений. Решение задание операции. Владеть приемами
системы линейных Устный опрос решение задач.
уравнений в матричной выполнение домашних работ по
форме. теме
Решение линейных Решение задач Знать основные логические
3. уравнений по формулам операции. Владеть приемами
Крамера . Решение решение задач.
линейных уравнений выполнение домашних работ по
методом Гаусса. теме
Комплексные числа. Решение задач Владеть понятием о множеством
4. Действия над его элементов
комплексными числами в Знать действие над
алгебраической форме. комплексными числами.
Производная. Общее Индивидуальное Владеть понятием о основные
5. правило нахождения задание элементарные функции.
производной. Правила и Устный опрос Знать основные логические
формулы операции. Владеть приемами
дифференцирования решение задач.
элементарных функций. выполнение домашних работ по
Правила теме
дифференцирования
суммы, произведения и
частного.
Неопределенный Индивидуальное Уметь решение задачи о
6. интеграл. Основные задание вычисление определенных
свойства неопределенного Устный опрос интегралов
интеграла.
Интегрирование
подстановкой и по частям.
Определенный интеграл.
Вычисление
определенных интегралов.
Элементы комбинаторики Индивидуальное Владеть приемами решение
7. и теория вероятностей. задание задач.
Перестановка.Сочетание.Р Устный опрос выполнение домашних работ по
азмешение теме.
Порядковый номер темы Количество академических часов
дисциплины Очная
№ п/п (Тема №) прак
лек
т. Срс
1. Тема 1 2 2 4
2. Тема 2 2 2 4
3. Тема 3 2 2 4
4. Тема 4 2 2 2
5. Тема 5 2 2 2
6. Тема 6 2 2 4
7. Тема 7 2 2 4
8. Тема 8 2 2 4
Общий объем учебной нагрузки (в
16 16 28
часах)
Всего часов: 60

3.Тематика курсовой работы ( не предусмотрена ).

4.Тематика, формы выполнения и критерии оценки СРС


Раздел программы 1-модуль Кол-во
Тема. часов
Матрицы. Действия над СРС №1.
матрицами и векторами . Подготовка презентации или реферата по 4
Определитель матрицы. теме «Матрицы. Действия над матрицами и
векторами» (работа со справочной и
дополнительной литературой, Интернет
источниками).

Свойства определителей и
их вычисления. Обратная СРС №2
матрица. Обращение Подготовка презентации или реферата по
матриц второго и третьего теме «Определитель. Свойства определителей» 2
порядков. (работа со справочной и дополнительной
литературой, Интернет источниками).

Системы линейных СРС №3


уравнений. Решение Подготовка презентации или реферата по
системы линейных теме «Решение системы линейных уравнений в 4
уравнений в матричной матричной форме.» Решение задач.
форме. Выполнение домашних работ по теме
Решение линейных СРС №4 Подготовка презентации или
уравнений по формулам реферата по теме «Решение линейных
Крамера . Решение уравнений по формулам Крамера . Решение 4
линейных уравнений линейных уравнений методом Гаусса.»
методом Гаусса.
Действительные и СРС №5 2
комплексные числа. Подготовка презентации или реферата по
Множества. Числовые теме «Комплексные числа» Решение задач.
множества. Множества Выполнение домашних работ по теме
действительных чисел.
Комплексные числа.
Действия над
комплексными числами в
алгебраической форме.
Элементы комбинаторики и СРС №6
теория вероятностей. 1.Составление конспекта по теме
Перестановка. Размещения.
Сочетание. «Элементы комбинаторики и теория

вереятностей» (работа со справочной и


4
дополнительной литературой, Интернет

источниками).

Правила СРС №7
дифференцирования суммы, Решение типовых задач
произведения и частного.
4
Правило
дифференцирования
сложной функции.
Неопределенный интеграл. СРС №8 4
Основные свойства Подготовка презентации или реферата по
неопределенного интеграла. теме «Неопределенный интеграл.» Решение задач.
Интегрирование Выполнение домашних работ по теме
подстановкой и по частям.
Определенный интеграл.
Вычисление определенных
интегралов. Вычисление
площадей плоских фигур.
Итого: 28

Параметры оценивания Количество баллов


Максимальное
Понимание содержания СРС(реферата ,эссе, 5
доклада и др.)через четкую формулировку
целей и задач.
Выполнение индивидуального домашнего 5
задание
Всего 10

4.2. Критерий оценки текущего и итогового контроля


Текущий контроль Итоговый
контроль
60 % 40 %
1 контрольная точка (модуль)0-30баллов 2 контрольная точка (модуль)0- 0-40
30баллов баллов

Оперативн Рубежный Самостоятель Оператив Рубежный Самостоятел


ый контроль ная работа ный контроль ьная работа
контроль контроль
0-10 0-10 0-10 баллов 0-10 0-10 0-10 баллов
баллов баллов баллов баллов
0-60 баллов 0-40
баллов
Максимальное количество баллов по дисциплинам – 100 баллов

4.3.Форма критерии оценки итогового контроля –экзамена (письменная работа)


Письменный опрос
Критерий оценки Кол-во
баллов
1. Понимание содержания 0 – 10
2. Наличие плана письменного ответа 0 – 10
3. Наличие теоретических знаний и практических умений 0 – 10
при выполнении письменного задания
4. Оформление работы 0–5
5. Приведение примеров 0–5

5.Критерии для оперативного, рубежного и итогового контроля достижений


студентов
Описать критерии и требования к студентам для оценки уровня их знаний и компетенций,
разработанные преподавателем (для каждого модуля и итогового контроля) с учетом специфики
дисциплины и общих требований.
Оперативный контроль – контроль за всеми видами аудиторной и внеаудиторной работы
обучающихся по дисциплинарному модулю, результаты которой оцениваются до рубежного
контроля.
Рубежный контроль – проверка полноты знаний, умений и навыков по материалу модуля в
целом. Рубежный контроль осуществляется два раза в семестр на учебных занятиях согласно
утвержденному графику проведения рубежного контроля.
Текущий контроль - проверка полноты знаний, умений и навыков по материалам двух модулей
в течение семестра, который состоит из оперативного, рубежных контролей и проверки
самостоятельной работы.
Итоговый контроль – форма контроля, проводимая по завершении изучения дисциплины в
семестре.
Формы и методы проведения контроля текущей, рубежной, итоговой успеваемости по
дисциплинам определяется учебно-методической комиссией факультета (или педагогическим
советом колледжа):
 устный опрос;
 письменный опрос;
 тестирование (бланочное или компьютерное).

Рейтинг знания студентов оценивается по 100- балльной шкале.


Результаты обучения студентов, его рейтинг оценивается по 100 – балльной шкале.
Рейтинговая оценка текущего и итогового контроля составляет не более 60% (60 баллов – это 30 на
1 модуль и 30 баллов на 2 модуль,), оставшиеся 40% (40 баллов) составляет итоговый контроль по
нижеследующей следующей схеме 1.
5.1. Контрольные вопросы и задания для проведения рубежного контроля
Максимальное время рубежного контроля до 30 минут;
Максимальный балл рубежного контроля-10 баллов;

Задания на 1 модуль
по дисциплине: «Профессиональная математика»
I вариант
1. Решить систему линейных алгебраических уравнений:
-по формулам Крамера;
-методом Гаусса;
-матричным способом;

{
7 x−3 y +5 z =32
5 x +2 y+ z=11
2 x− y +3 z=14 ;

2
053
2. Найдите × 1
123
0
3. Найдите площадь треугольника с вершинами в точках А (3;5;4), В (1;-1;3), С (7;2;-6)
4. а ̅ =(5;4;0), ̅b=(-1; 0;1). Найдите: ̅а×̅b

Задания на 1 модуль
по дисциплине: «Профессиональная математика»
II вариант
1. Решить систему линейных алгебраических уравнений:
-по формулам Крамера;
-методом Гаусса;
-матричным способом;

{
3 x+2 y +2 z=1
x +3 y + z=0
5 x +3 y + 4 z=1 ;

3
13 2
2. Найдите × 1
524
0
Задания на 2 модуль
по дисциплине: «Профессиональная математика»
I вариант
1. Сколькими способами можно рассадить 5 человек за столом?
2. Найти производные следующих функций.
а) y=5 x 4 −3 √7 x 3+7/ x 5 +¿4 б) y=ln ( x 2+2 x)
3. Вычислить определенные интегралы.
2

∫ (2 x2 + x24 ) dx
1

Задания на 2 модуль
по дисциплине : «Профессиональная математика»
II вариант
1. Сколько различных слов из 6 букв можно составить из букв в слове Жумгал?
2. Найти производные следующих функций.
а) y=( x 4 +1 ¿ ( x 4−1 )б) y=cos ln(1−x 2 )
3. Вычислить определенные интегралы.
3

∫ (x 2+ x23 )dx
1

5.2. Контрольные вопросы итогового контроля

Задания на итоговый контроль


по дисциплине: «Профессиональная математика»
I вариант
1. Найти линейные комбинации матриц:
2А+3В, где А= ( 1 23
0 1−1) (
, В= )
−2 3 0
21 1
.
2. Решите систему методом Крамера и матричным методом:

{
2 x−4 y +3 z=1
x −2 y + 4 z=3
3 x− y +5 z=2
3. Найти производные следующих функций:
3
x
а) y= −2 x 2+ 10 , б) y= x 2 cos x
3
4. Вычислить определенные интегралы:
2

(
1
2
)2
а) ∫ 2 x + 4 dx
x
б) ∫ е sin xdx
cosx

Задания на итоговый контроль


по дисциплине: «Профессиональная математика»
II вариант
1. Найти линейные комбинации матриц:
4А-5В, где А= (
2−1 0
3 4−2 )
, В= (
312
−2 1 3
.)
2. Решите систему методом Крамера и матричным методом:
{
x +2 y+3 z=5
2 x − y−z=1
x +3 y +4 z=6
3.Найти производные следующих функций:
а) y=( x 5 +3 x−1) 4 , б) y=ln ( x 2+2 x)
4. Вычислить определенные интегралы:
2

(1
)2 2
а) ∫ 3 x + 2 dx
x
б) ∫ cos ² xdx

Контрольные вопросы итогового контроля.


1. Что такое множество?
2. Пустое, конечное и бесконечное множество.
3. Объединение множеств.
4. Пересечение множеств.
5. Разность множеств.
6. Диаграмма Эйлера- Венна.
7. Свойства объединения и пересечения множеств
8. Число элементов объединения и разности двух конечных множеств
9. Прямоугольная система координат.
10. Расстояние между двумя точками.
11. Деления отрезка в данном отношении.
12. Площадь треугольника
13. Уравнения прямых.
14. Общее уравнение прямых.
15. Уравнение прямых с угловым коэффициентом.
16. Расстояние от точки до прямой.
17. Условие параллельности и перпендикулярности прямых.
18. Постоянные и переменные
19. Представление функции формулой и таблицей
20. Обозначение функций. Координаты. Графическое представление функций
21. Монотонная функция. Ограниченная и неограниченная функции.
22. Непрерывная и разрывная функции.
23. Чётная и нечётная функции.
24. Предел последовательности и функции.
25. Свойство пределов.
26. Первый и второй замечательный предел.
27. Производная.
28. Геометрический смысл производной.
29. Уравнение касательной.
30. Механический смысл производной.
31. Основные свойства производных и дифференциалов
32. Производные элементарных функций
33. Основные определения и свойства интегралов
34. Свойства неопределенного интеграла:
35. Таблица интегралов
36. Методы интегрирования неопределенных интегралов
37. Определенный интеграл
38. Формула Ньютона-Лейбница
39. Объем тела вращения
40. Методы интегрирования определенного интеграла.
Задачи для самостоятельной работы студентов
Решение задач по темам: операции над матрицами, вычисление определителей,
миноры и алгебраические дополнения, вычисление обратной матрицы, ранг матрицы,
применения в экономике.

1. Найти линейные комбинации матриц

A=¿ ( 1 2 3 ¿ ) ¿ ¿¿ B=¿ (−2 3 0 ¿ ) ¿ ¿¿


а) 2А+3В, где ¿ , ¿ .

A=¿( 2−1 0¿) ( 3 4−2¿) ¿¿¿ B=¿ ( 3 1 2¿ )(−2 1 3¿ )¿ ¿¿


б) 4А – 5В, где ¿ , ¿ .
A=¿( 7−2 3−4¿) ( 0 2 1−1¿) ¿¿¿ B=¿(2 −1−3 1¿)(7 −1 0 4¿) ¿¿¿
в) 3А+4В, где ¿ , ¿ .
2. Найти произведения АВ и ВА (если это возможно).

A=¿ ( 3 −2 ¿ ) ¿ ¿ ¿
а) ¿

B=¿(3 4 5¿)(6 0−2¿)¿¿¿


A=¿ ( 1 2 3 ¿ ) ¿ ¿¿
б) ¿ ¿, .

A=¿ ( 1 2 ¿ ) ¿ ¿ ¿
в) ¿ .
3. Предприятие производит продукцию трех видов и использует сырье двух типов. Нормы затрат

сырья на единицу продукции каждого вида заданы матрицей


A= 2 1 3 .
1 3 4 Стоимость ( )
единицы сырья каждого типа задана матрицей В=(10 15). Каковы общие затраты
предприятия на производство 100 единиц продукции первого вида, 200 единиц продукции
второго вида и 150 единиц продукции третьего вида?

4. Вычислите определители второго порядка.


|3 −2¿|¿¿¿ |2 3¿|¿ ¿¿ |3 −2¿|¿ ¿¿ |√ а −1 ¿|¿ ¿¿ | sinα cosα ¿|¿ ¿¿
¿ , б) ¿ , в) ¿ , г) ¿ , д) ¿
а)
5. Вычислите определители третьего порядка.

|3−2 1¿| −2 2 3¿|¿¿¿


|1 2 0¿| 0 1 3¿|¿¿ |2−1 3¿| −2 3 2¿|¿¿¿ |а 1 а¿|−1 а 1¿|¿¿¿
а) ¿ б) ¿ , , в) ¿ , г) ¿

A=(
6 3 ) . Найти все миноры и алгебраические дополнения.
5 7
6. Дана матрица

( )
5 −2 3
А= 2 −1 0
7. Дана матрица 1 1 2 . Найти все миноры и алгебраические дополнения.

8. Вычислите определители, используя разложение по строке, либо по столбцу.


1 2 3 2
2 5 7 1 0 −1 0 1 −2 3
|0 1 2 | |7 −7 21 | | |
−1 −1 0 4
а) 0 2 4 ; б) 4 8 12 ; в) 2 3 −2 5 .
−1
9. Найти обратную матрицу A

( ) ( )
1 2 1 5 −2 3

а)
A= (56 73 ) ; б)
А= 2 4 2
1 1 2 ; в)
А= 2 −1 0
1 1 2 .

Решение задач по темам: системы линейных уравнений и нахождение их решений


методами Крамера , Гаусса, обратной матрицы.

1.Решите системы методом Крамера и матричным методом :

а)
{ 2 х −4 y + 3 z = 1 ¿ { x −2 y + 4 z =3 ¿ ¿ { х + 2 y + 3 z =5 ¿ { 2 x − y − z =1 ¿ ¿ { х + y + z =a ¿ { x − y + z =b ¿ ¿
б) в)

2. Методом Гаусса решите следующие линейные системы:

а)
{х1+х2+х3=8¿{5x1+2х2+4х3=14¿¿ ¿ б)
{х1+2х2−х3=2¿{2x1−6х2+х3=−11¿¿ ¿
в)
{ x1+3x2−x3=2 ¿ {2 x1−x2+x3+3x4=14 ¿ ¿¿¿¿
г)
{2 x 1 +7 x 2 +3 x 3 + x 4 = 5 ¿ { x 1 + 3 x 2 + 5 x 3 − 2 x 4 = 3 ¿ { x 1 + 5 x 3 − 9 x 3 + 8 x 4 = 1 ¿ ¿
д)
{x1−2x2+x4=−3¿{3x1−x2−2x3=1¿{2x1+x2−2x3−x4=4 ¿ ¿ {x1+2x2+3x3=6¿{2x1−3x2+x3=0¿{3x1−2x2+4x3=5 ¿ ¿
е) .

3. Найти производные следующих функций:

2 3
y= −3
−3 √ x +7 / x +4 ;
7
a) һy=5 x
4 3 5
b) √ x √ x c) y=( x 4 +1 ) / ( x 4 −1 ) ;

d)
y=
1+ln x
x e) y=( sin x ) / ( x +1 )
2 3
f)
y=
√ cos 2 x +1
sin 2 x+ 1 g) y=( x +3 x−1 ) ;
5 4
h)

3 arctg √1+ x2 tg 3 5 x
y= √ x +sin x k) y=( 2 −tg x )
3 4 4 x4 4
m) y=e n) y=3 √
3
o) y= (4+3 x )
2

y=
1 1+ √ x 2 +1
y=ln
q) (1−x 2 )5
2
r) y=ln( x +2 x) s) y=cosln(1−x ) t)
2
x √
5
u) y= x +x √ x
3

x
v) y=e tg 4 x ; w) y=x cos x z) y=e
2 x
√ 1−e 2 x +arcsin e x
4. Используя логарифмическую производную найти производные следующих функций:
1
x2 tg 2 x
e) y=( x +1 )
cos 5 x 3
x
a) y=(sin x ) b) y=x c) y=(cos x )
x
d) y= ( sin3 x ) .

''
5. Найти производную второго порядка y функций
a) y=( 1+ 4 x ) arctg 2 x b) y=( x +1)ln(1+x )
2 2 2

3 2
6. Найти значения производных любого порядка функции y=x −5 x +7 x−2 в точке x=2

2x
7.Удовлетворяет ли функция y=C 1 e +C 2 e 3 x при любых постоянных C 1 и C 2 уравнению
y ' ' −5 y ' +6 y =0 .

Решение задач по теме: вычисление интеграла.


Найти указанные неопределенные интегралы и результаты проверить
дифференцированием.

1)Методом непосредственного интегрирования


a)
(
∫ ex 1− x 2
e− x
) dx
b)
∫ (√1
3 2
x

1
x√x ) dx
c)
∫ x−2x3 dx

2) Методом замены переменной
e√
x x
∫ dx
∫ tgx dx ∫ e
cos x
sin x dx
∫ x dx
d) √
2
a) x +1 b) c)
dx x+1
∫ √ 1+ln x
x
dx ∫
√ x 2+2x+3 g) ∫ x2+3dxx+3 ∫ √3 3 x+1 dx
e) f) h)
sin √ x
dx ∫ 6
x 2 dx
1−2 x3 x dx ∫ √ x
6 2
k)∫ 2 3
sin x cos x dx
m)∫ √ n)
3
o) x +2 x +3
3) Методом интегрирования по частям
x


a) x ln x−1 dx b)
( ) √ x ln x dx c) x e ∫ ∫ 2 2
dx
4) Вычислить определенные интегралы.
e3 √3
dx
∫ ∫ x 5 √ 1+ x2 dx .
2

a)

1
( 2 x2 +
2
x4
dx .
)b) 1
x √ 1+ ln x
.
c) 1
1 8
∫ xe−x dx ∫ √xdx
1+x
d) 0 e) 3
5) Вычислить несобственные интегралы.
+∞ ∞ ∞
dx dx
∫ 2
−∞ x +1
∫ x ln x
, ( a>1 ) ∫ sin xdx
a) b) a c) 0
6. Рекомендуемая литература:

ОСНОВНАЯ ЛИТЕРАТУРА
1.Туганбаев У.М.,Туганбаев М.М. «Высшая математика» Учебник для ВУЗои -2005.
2.Дмитрий Письменный.»Корспект лекций по высшей МАТЕМАТИКЕ» 1часть,
Москва АЙРИС ПРЕСС 2017г.
3. Усубакунов Т.»Дифференциалдык тендемелер».-Ф:Жогорку математика.,1988
4. И.Л. Соловейчик, В.Т. Лисичкин. Сборник задач по математике для техникумов.
5.Высшая математика. Учебное пособие.В.Л. Ким,В.Бостериева,Е.С.Федорова.Б.1998
6. Минорский В.П., Сборник задач по высшей математике.
7.Демидович Б. П. Сборник задач и упражнений по математическому анализу .-М.:
Наука, 1990.-624с.

ДОПОЛНИТЕЛЬНАЯ ЛИТЕРАТУРА

1. Тарасов Н.П. Курс высшей математики. -М: Наука, 1975.-448 с.


2. Гроссман С., Тернер Дж. Математика для биологов. -М.: Высшая школа, 1983.
3. Фихтенгольц Г. М. Основы математического анализа. Т.1.-М.: Наука, 1968.-440с.
4. Пискунов Н.С. Дифференциальное и интегральное исчисления. М.: Наука, 1978.-575с.
5. Шифр № В11/Б-42 Бекбоев И.Б. Жогорку математиканын жалпы курсу. Бишкек. 2000
6. Шифр № В11/0579 Омурбаев Н.Т. Жогорку математиканын жалпы элементтери. Бишкек
2012
7. Шифр № кырг.В16/Э-40 Урдалетова А.Б, Йылмаз Ж. Экономикалык жана коомдук
илимдер учун математика . Бишкек 2010.

Электронные ресурсы:
1.https//clck/ru/34Srat
2 file:///E:/проф.матем/матем.книга%20для%20СПО_Глотова_Самохвалова-2016г.и..pdf

www.okuma.kg
3 https://fileskachat.com/download/6647_587665e51b67eb5404f4da219e6cc079.html
4 http://www.bymath.net/ - элементарная математика
5 http://graphfunk.narod.ru — графики элементарных функций
6 http://window.edu.ru/window — информационная система «Единое окно доступа к
образовательным ресурсам» с обширной библиотекой по основным
разделам
7 математики.
7.http://www.exponenta.ru/ - образовательный математический сайт

Кыргызский национальный университет им. Ж. Баласагына


Профессиональный колледж

Дисциплина: «Профессиональная математика»


ЭКЗАМЕНАЦИОННЫЙ БИЛЕТ №1 Критерий оценки
1.Матрица. Определение обратной матрицы. 10 баллов
2.Определение комплексного числа .
10 баллов

( )( )
301 2
3.Найдите произведение матриц : −1 0 5 ∙ 3
11 2 1 20 баллов
Итого за задание 40 баллов
Составитель: Джолумбекова М.Т.
Зав.отд.: Куртубаева Н.К.

Кыргызский национальный университет им. Ж. Баласагына


Профессиональный колледж

Дисциплина: «Профессиональная математика»


ЭКЗАМЕНАЦИОННЫЙ БИЛЕТ №2 Критерий оценки
1. Элементы комбинаторики. 10 баллов
2. Определитель матриц.Виды матриц. 10 баллов
3.Найти производных функций .
у= сos(2+3x) 20 баллов

Итого за задание 40 баллов


Составитель: Джолумбекова М.Т.
Зав.отд.: Куртубаева Н.К.

Кыргызский национальный университет им. Ж. Баласагына


Профессиональный колледж
Дисциплина: «Профессиональная математика»
ЭКЗАМЕНАЦИОННЫЙ БИЛЕТ №3 Критерий оценки
1.Предел последовательности и функции. 10 баллов
2.Производная. 10 баллов
3.Найти производная функций: 20 баллов
Y=sinx+2x

Итого за задание 40 баллов


Составитель : Джолумбекова М.Т.
Зав.отд.: Куртубаева Н.К.

Кыргызский национальный университет им. Ж. Баласагына


Профессиональный колледж
Дисциплина: «Профессиональная математика»
ЭКЗАМЕНАЦИОННЫЙ БИЛЕТ №4 Критерий оценки
1.Определения множества , пустого множества. 10 баллов
Подмножества.
2. Определитель. Свойства определителей. 10 баллов
3.Методом замены переменной
x 20 баллов
∫ x2+1 dx
Итого за задание 40 баллов
Составитель: Джолумбекова М.Т.
Зав.отд.: Куртубаева Н.К.

Кыргызский национальный университет им. Ж. Баласагына


Профессиональный колледж
Дисциплина: «Профессиональная математика»
ЭКЗАМЕНАЦИОННЫЙ БИЛЕТ №5 Критерий оценки
1.Комплексные число. Действия над 10 баллов
комплексными числами. 10 баллов
2.Свойство определителей. 20 баллов
''
3.Найти производную второго порядка y функций
y=( 1+ 4 x 2 ) arctg 2 x
Итого за задание 40 баллов
Составитель: Джолумбекова М.Т.
Зав.отд.: Куртубаева Н.К.

Кыргызский национальный университет им. Ж. Баласагына


Профессиональный колледж
Дисциплина: «Профессиональная математика»
ЭКЗАМЕНАЦИОННЫЙ БИЛЕТ №6 Критерий оценки
1.Миноры и алгебраические дополнения. 10 баллов
2.Определенные интегралы. 10 баллов
''
3.Найти производную второго порядка y функций
y=( x 2 +1)ln(1+x 2 ) 20 баллов

Итого за задание 40 баллов


Составитель: Джолумбекова М.Т
Зав.отд.: Куртубаева Н.К.

Кыргызский национальный университет им. Ж. Баласагына


Профессиональный колледж
Дисциплина: «Профессиональная математика»
ЭКЗАМЕНАЦИОННЫЙ БИЛЕТ №7 Критерий оценки
1.Дифференциал функции. 10 баллов
2.Что называется матрицей, матрицей- строкой, 10 баллов
матрицей- столбцом, вектором?
3. Вычислить определенные интегралы. 20 баллов
√3
∫ x √5 1+ x ² d x
1

Итого за задание 40 баллов


Составитель: Джолумбекова М.Т.
Зав.отд.: Куртубаева Н.К.

Кыргызский национальный университет им. Ж. Баласагына


Профессиональный колледж
Дисциплина: «Профессиональная математика»
ЭКЗАМЕНАЦИОННЫЙ БИЛЕТ №8 Критерий оценки
1.Комплексные числа. Тригонометрическая форма 10 баллов
комплексного числа.
2. Что называется минором? 10 баллов
3. Вычислить определенные интегралы. 20 баллов
2

( )
∫ 2 x2 + 24 dx .
1 x
Итого за задание 40 баллов
Составитель: Джолумбекова М.Т.
Зав.отд: Куртубаева Н.К.

Кыргызский национальный университет им. Ж. Баласагына


Профессиональный колледж
Дисциплина: «Профессиональная математика»
ЭКЗАМЕНАЦИОННЫЙ БИЛЕТ №9 Критерий оценки
1. Какая матрица называется обратной по отношению к 10 баллов
данной?
2. Действия над комплексными числами. 10 баллов
3. Произвести умножение комплексных чисел: 20 баллов
(−2+3 i ) (3+5 i)
Итого за задание 40 баллов

Составитель: Джолумбекова М.Т.


Зав.отд.: Куртубаева Н.К.

Кыргызский национальный университет им. Ж. Баласагына


Профессиональный колледж
Дисциплина: «Профессиональная математика»
ЭКЗАМЕНАЦИОННЫЙ БИЛЕТ №10 Критерий оценки
1. Действия над матрицами. 10 баллов
2. Расстояние между двумя точками. 10 баллов
3.Найти производные функций. 20 баллов
4
y=3 x ³+5 √ х − 3
3 5

х
Итого за задание 40 баллов
Составитель: Джолумбекова М.Т.
Зав.отд.: Куртубаева Н.К.

Кыргызский национальный университет им. Ж. Баласагына


Профессиональный колледж
Дисциплина: «Профессиональная математика»
ЭКЗАМЕНАЦИОННЫЙ БИЛЕТ №11 Критерий оценки
1. Элементы комбинаторики. 10 баллов
2. Таблица интегралов. 10 баллов
3. Найти производную функцию: 20 баллов
2
2 x −6
y=
x−2

Итого за задание 40 баллов


Составитель: Джолумбеова М.Т.
Зав.отд.: Куртубаева Н.К.

Кыргызский национальный университет им. Ж. Баласагына


Профессиональный колледж
Дисциплина: «Профессиональная математика»
ЭКЗАМЕНАЦИОННЫЙ БИЛЕТ №12 Критерий оценки
1. Правила дифференцирования алгебраической суммы, 10 баллов
произведения и частного.
2. Действия над матрицами. 10 баллов
3. Найти производную функцию: 20 баллов
x3
y=
x 2 −4 ;
Итого за задание 40 баллов
Составитель: Джолумбекова М.Т.
Зав.отд.: Куртубаева Н.К..

Кыргызский национальный университет им. Ж. Баласагына


Профессиональный колледж
Дисциплина: «Профессиональная математика»
ЭКЗАМЕНАЦИОННЫЙ БИЛЕТ №13 Критерий оценки
1. Неопределенный интеграл. 10 баллов
2. Дайте определение предела функции в точке.
3. Как определить частное значение функции? Проверьте, 15 баллов
правильно ли вычислено f ( 2 )=5, если f ( x )=x 3− x+1 20 баллов
Итого за задание 40 баллов
Составитель: Джолумбекова М.Т.
Зав.отд.: Куртубаева Н.К.

Кыргызский национальный университет им. Ж. Баласагына


Профессиональный колледж
Дисциплина: «Профессиональная математика»
ЭКЗАМЕНАЦИОННЫЙ БИЛЕТ №14 Критерий
оценки
1. Комплексные числа. 10 баллов
2. Элементы комбинаторики. 10 баллов
3.Найти производные: 20 баллов
3
2 n −3 n+5
у=❑ 3
3+5 n−6 n
Итого за задание 40 баллов
Составитель: Джолумбекова М.Т.
Заав.отд.: Куртубаева.Н.К.

Кыргызский национальный университет им. Ж. Баласагына


Профессиональный колледж
Дисциплина: «Профессиональная математика»
ЭКЗАМЕНАЦИОННЫЙ БИЛЕТ №15 Критерий
оценки
1. Определение комплексные число. 10 баллов
2. Общее правило нахождения производной. 10 баллов
3. Найти производные: 20 баллов
1
у ( 1+2 )
n n

Итого за задание 40 баллов


Составитель : Джолумбекова М.Т.
Зав.отд: Куртубаева Н.Т.

Кыргызский национальный университет им. Ж. Баласагына


Профессиональный колледж
Дисциплина: «Профессиональная математика»
ЭКЗАМЕНАЦИОННЫЙ БИЛЕТ №16 Критерий оценки
1. Неопределенный интеграл и его свойства. 10 баллов
2. Определение матрицы. Виды матриц. 10 баллов
3. Решите системы методом Крамера и матричным методом : 20 баллов

{ 2 х −4 y + 3 z = 1 ¿ { x −2 y + 4 z =3 ¿ ¿
Итого за задание 40 баллов
Составитель: Джолумбекова М.Т.

Зав.отд.: Куртубаева .Н.К.

Кыргызский национальный университет им. Ж. Баласагына


Профессиональный колледж
Дисциплина: «Профессиональная математика»
ЭКЗАМЕНАЦИОННЫЙ БИЛЕТ №17 Критерий оценки
1. Методы интегрирования определенного интеграла. 10 баллов
2. Виды матрицы. 10 баллов
3. Найти линейные комбинации матриц 20 баллов

A=¿ ( 1 2 3 ¿ ) ¿ ¿¿ B=¿ (−2 3 0 ¿ ) ¿ ¿¿


2А+3В, где ¿ , ¿ .
Итого за задание 40 баллов
Составитель : Джолумбекова М.Т.
Зав.отд.: Куртубаева Н.К.

Кыргызский национальный университет им. Ж. Баласагына


Профессиональный колледж
Дисциплина: «Профессиональная математика»
ЭКЗАМЕНАЦИОННЫЙ БИЛЕТ №18 Критерий оценки
1. Производная. 10 баллов
2. Основные определения и свойства интегралов. 10 баллов
3. Найти линейные комбинации матриц 20 баллов

A=¿( 2−1 0¿) ( 3 4−2¿) ¿¿¿ B=¿( 3 1 2¿)(−2 1 3¿)¿¿¿


4А – 5В, где ¿ , ¿ .
Итого за задание 40 баллов
Составитель: Джолумбекова М.Т.
Зав.отд .: Куртубаева Н.К.

Кыргызский национальный университет им. Ж. Баласагына


Профессиональный колледж
Дисциплина: «Профессиональная математика»
ЭКЗАМЕНАЦИОННЫЙ БИЛЕТ №19 Критерий оценки
1. Таблица интегралов 10 баллов
2. Дифференциал функции. 10 баллов
3. Найти производные 20 баллов
2
x −7 x +10
у=

8−x ³

Итого за задание 40 баллов


Составитель: Джолумбекова М.Т..
Зав.отд. : Куртубаева Н.К.

Кыргызский национальный университет им. Ж. Баласагына


Профессиональный колледж
Дисциплина: «Профессиональная математика»
ЭКЗАМЕНАЦИОННЫЙ БИЛЕТ №20 Критерий оценки
1. СЛАУ.Метод Гаусса. 10 баллов
2. Основные определения и свойства интегралов. 10 баллов

3. Найти производные функций. 20 баллов


У= (2х-234)/(3+2х)

Итого за задание 40 баллов


Составитель: Джолумбекова М.Т.
Зав.отд. : Куртубаева Н.К.
МИНИСТЕРСТВО ОБРАЗОВАНИЯ И НАУКИ КЫРГЫЗСКОЙ РЕСПУБЛИКИ
КЫРГЫЗСКИЙ НАЦИОНАЛЬНЫЙ УНИВЕРСИТЕТ
имени Ж. БАЛАСАГЫНА
ПРОФЕССИОНАЛЬНЫЙ КОЛЛЕДЖ ОТДЕЛЕНИЕ ПЕРЕВОДЧЕСКОЕ ДЕЛО

SYLLABUS

по дисциплине: Профессиональная математика

Специальность подготовки 050720 -Переводческое дело.


(указать шифр и название) 100201 - Туризм

Учебную программу (Syllabus) дисциплины подготовила преподаватель специальных дисциплин


Профессионального колледжа Джолумбекова М.Т.
(должность, Ф.И.О., подпись)

Одобрено на заседании предметно-цикловой комиссии

Протокол №_____ от «______»______________20__ г.


___________________________________________
(Ф.И.О. председателя УМК, подпись)

Сведения о преподавателе: Джолумбекова Махабат Темиркуловна преподаватель


естественных дисциплин отделение Переводческое дело Профессионального колледжа КНУ
им. Ж.Баласагына.
Разработаны учебные программы по дисциплинам «Профессиональная математика»,
« Математика».

Контактная информация:
Кыргызский национальный университет им. Ж. Баласагына отдела переводческое дело
Профессиональный колледжа.
Адрес: ул. Калык-Акиева 85, корпус 6А
m.jolumbekova@gmil.com
Контактный 0709333800

Дисциплина « Профессиональная математика» относиться к циклу специальных


дисциплин для студентов специальности 050720- « Переводческое дело » 100201 «Туризм»

 Дисциплина «Профессиональная математика» по объему составляет 2 кредитов (всего


60 часов), из них 32 аудиторных занятий (16 лекций и 16 практических) и 28 часов
самостоятельной работы студента.
 «Профессиональная математика» изучают студенты 2 курса на базе 9 класса в 3
семестре, и студенты 1 курса на базе 11 класса в 1 семестре. Продолжительность учебы
составляет 16 недель.
 Пререквезиты дисциплины:

-линейная алгебра, прикладная математика, исследование операций, теория


оптимизации, теория вероятности и математическая статистика.
 Постреквезиты дисциплины:

-линейная алгебра, прикладная математика.

Цели и задачи дисциплины


Целью изучения дисциплины «Профессиональная математика» является:
 дать студентам математические знания необходимые для изучения специальных,
инженерных дисциплин и развития навыков, требуемых для применения
математических методов в практике инженера.
 обучение основным математическим понятиям и методам, необходимым для анализа
и моделирования экономических проблем при поиске рациональных решений в
сложных условиях;
 формирование математической культуры студентов.

Задачи дисциплины:
• Выполнять арифметические действия над числами, сочетая устные и письменные приемы;
• сравнивать числовые выражения;
• находить значения корня, степени, логарифма, тригонометрических выражений на основе
определения, используя при необходимости инструментальные средства;
• пользоваться приближенной оценкой при практических расчетах ;
• выполнять преобразования выражений, применяя формулы, связанные со свойствами степеней,
логарифмов, тригонометрических функций;

Расписание аудиторных занятий


Время Понедельник Вторник Среда Четверг Пятница
13.00-
14.20

14.30-
15.55

16.00-
17.20

Расписание аудиторных занятий

Врем Понедельни Вторник Среда Четверг Пятница


я к
13.00-
14.20

14.30-
15.55

16.00-
17.20

Содержание и трудоемкость дисциплины


№ Количество академических часов
Очная
Название тем и их содержание
Лек. Практ. СРС
1 модуль
Элементы линейной алгебры. Матрицы. Действия
1 над матрицами и векторами .
2 2 4

2 Определитель матрицы. Свойства определителей и


2 2 2
их вычисления. Обратная матрица. Обращение
матриц второго и третьего порядков.
Элементы линейной алгебры. Системы линейных
3 уравнений. Решение системы линейных уравнений в 2 2 2
матричной форме.
СЛАУ.Метод Крамера.
4 2 2 4
СЛАУ. Метод Гаусса.
2 модуль
Элементы комбинаторики и теория вероятностей.
5 2 2 4
Перестановка.Размещения. Сочетание.
Производная. Общее правило нахождения
производной. Правила и формулы
6 2 2 4
дифференцирования элементарные
функций.Дифференциал функции.

Комплексные числа. Действия


7 2 2 4
над комплексными числами в алгебраической
форме.
Неопределенный интеграл. Основные свойства
неопределенного интеграла. Интегрирование
8 подстановкой и по частям. Определенный интеграл. 2 2 4
Вычисление определенных интегралов. Вычисление
площадей плоских фигур.
Общий объем учебной нагрузки (в часах) 16 16 28
Всего часов:60 32 28

Рейтинговая оценка текущего и итогового контроля


Текущий контроль Итоговый
контроль

60% 40%
1 контрольная точка 2 контрольная точка
0-40
Оперативный Рубежный Самостоятельная Оперативный Рубежны Самостоятельная баллов
контроль контроль работа контроль й работа
контроль

0-10 баллов 0-10 0-10 баллов 0-10 баллов 0-10 0-10 баллов
баллов баллов

0-30 баллов 0-30баллов 0-40


баллов
0-60 баллов

Максимальное количество баллов по дисциплинам 100 баллов

1-яКонтрольная работа осуществляется на 8-й неделе


2-я Контрольная работа осуществляется на 16-й неделе
Итоговая контрольная работа (Экзамен) проводится на 17-й неделе

Шкала оценки результатов обучения студентов


Оценка по Оценка по
100 балльной традиционной
шкале системе
0-60 Неудовлетворительно
61-73 Удовлетворительно
74-86 Хорошо
87-100 Отлично

Требования преподавателя: отрабатывать пропущенные по уважительным причинам занятия в


согласованное с преподавателем время;
- активно участвовать в учебном процессе;
- самостоятельно заниматься по индивидуально заданным темам
Нормы академической этики:
Конфликтные ситуации должны открыто обсуждаться в учебных группах.

Контрольные вопросы итогового контроля.


1. Что такое множество?
2. Пустое, конечное и бесконечное множество.
3. Объединение множеств.
4. Пересечение множеств.
5. Разность множеств.
6. Диаграмма Эйлера- Венна.
7. Свойства объединения и пересечения множеств
8. Число элементов объединения и разности двух конечных множеств
9. Прямоугольная система координат.
10. Расстояние между двумя точками.
11. Деления отрезка в данном отношении.
12. Площадь треугольника
13. Уравнения прямых.
14. Общее уравнение прямых.
15. Уравнение прямых с угловым коэффициентом.
16. Расстояние от точки до прямой.
17. Условие параллельности и перпендикулярности прямых.
18. Постоянные и переменные
19. Представление функции формулой и таблицей
20. Обозначение функций. Координаты. Графическое представление функций
21. Монотонная функция. Ограниченная и неограниченная функции.
22. Непрерывная и разрывная функции.
23. Чётная и нечётная функции.
24. Предел последовательности и функции.
25. Свойство пределов.
26. Первый и второй замечательный предел.
27. Производная.
28. Геометрический смысл производной.
29. Уравнение касательной.
30. Механический смысл производной.
31. Основные свойства производных и дифференциалов
32. Производные элементарных функций
33. Основные определения и свойства интегралов
34. Свойства неопределенного интеграла:
35. Таблица интегралов
36. Методы интегрирования неопределенных интегралов
37. Определенный интеграл
38. Формула Ньютона-Лейбница
39. Объем тела вращения
40. Методы интегрирования определенного интеграла.

ПЕРЕЧЕНЬ ЛИТЕРАТУРЫ И СРЕДСТВ ОБУЧЕНИЯ


Основная учебная литературы
1.Туганбаев У.М.,Туганбаев М.М. «Высшая математика» Учебник для ВУЗои -2005.
2.Дмитрий Письменный.»Корспект лекций по высшей МАТЕМАТИКЕ» 1часть, Москва АЙРИС
ПРЕСС 2017г.
3. Усубакунов Т.»Дифференциалдык тендемелер».-Ф:Жогорку математика.,1988
Дополнительная литература:
1. И.Л. Соловейчик, В.Т. Лисичкин. Сборник задач по математике для техникумов.
2. Минорский В.П., Сборник задач по высшей математике.
3.Демидович Б. П. Сборник задач и упражнений по математическому анализу .-М.: Наука, 1990.-
624с.
КЫРГЫЗСКИЙ НАЦИОНАЛЬНЫЙ УНИВЕРСИТЕТ
имени Ж. БАЛАСАГЫНА
ПРОФЕССИОНАЛЬНЫЙ КОЛЛЕДЖ ОТДЕЛЕНИЕ ПЕРЕВОДЧЕСКОЕ ДЕЛО

КОНСПЕКТ ЛЕКЦИЙ

ДИСЦИПЛИНЫ

Профессиональная математика
Специальность «Переводческое дело», «Туризм»

Тема .1. Элементы линейной алгебры. Матрицы. Действия над матрицами. Свойства
операций над матрицами. Виды матриц.
Цели урока:
Образовательные – ознакомиться с понятиями матрицы, определителя матрицы, рангом
матрицы, научиться выполнять действия над матрицами.
Воспитательные - воспитание воли и упорства для достижения конечных результатов,
стремление к творческой деятельности.
Развивающие – развивать коммуникативные навыки во время практической работы;
организовывать собственную деятельность.

Матрицы (и соответственно математический раздел - матричная алгебра) имеют важное


значение в прикладной математике, так как позволяют записать в достаточно простой форме
значительную часть математических моделей объектов и процессов. Термин "матрица" появился
в 1850 году. Впервые упоминались матрицы еще в древнем Китае, позднее у арабских
математиков.
Матрицей A=Amn порядка m*n называется прямоугольная таблица чисел, содержащая m -
строк и n - столбцов.

Элементы матрицы aij, у которых i=j, называются диагональными и образуют главную


диагональ.
Для квадратной матрицы (m=n) главную диагональ образуют элементы a11, a22,..., ann .
Равенство матриц.
A=B, если порядки матриц A и B одинаковы и aij=bij (i=1,2,...,m; j=1,2,...,n)

Действия над матрицами.


1. Сложение матриц - поэлементная операция

2. Вычитание матриц - поэлементная операция


3. Произведение матрицы на число - поэлементная операция

4. Умножение A*B матриц по правилу строка на столбец (число столбцов матрицы А должно
быть равно числу строк матрицы B)

Amk*Bkn=Cmn причем каждый элемент сij матрицы Cmn равен сумме произведений элементов i-ой
строки матрицы А на соответствующие элементы j-го столбца матрицы B , т.е.

Покажем операцию умножения матриц на примере


5. Возведение в степень

m>1 целое положительное число. А - квадратная матрица (m=n) т.е. актуально только для
квадратных матриц

6. Транспонирование матрицы А. Транспонированную матрицу обозначают AT или A'

Строки и столбцы поменялись местами

Пример

Свойства операций над матрицами

A+B=B+A ,(A+B)+C=A+(B+C), λ(A+B)=λA+λB, A(B+C)=AB+AC, (A+B)C=AC+BC,

λ(AB)=(λA)B=A(λB), A(BC)=(AB)C, (A')'=A, (λA)'=λ(A)', (A+B)'=A'+B', (AB)'=B'A'.

Виды матриц

1. Прямоугольные: m и n - произвольные положительные целые числа


2. Квадратные: m=n
3. Матрица строка: m=1. Например, (1 3 5 7 ) - во многих практических задачах такая матрица
называется вектором

4. Матрица столбец: n=1. Например

5. Диагональная матрица: m=n и aij=0, если i≠j. Например


6. Единичная матрица: m=n и

7. Нулевая матрица: aij=0, i=1,2,...,m, j=1,2,...,n ,


8. Треугольная матрица: все элементы ниже главной диагонали равны 0.

Пример.

9. Симметрическая матрица: m=n и aij=aji (т.е. на симметричных относительно главной


диагонали местах стоят равные элементы), а следовательно A'=A

Например,

10. Кососимметрическая матрица: m=n и aij=-aji (т.е. на симметричных относительно главной


диагонали местах стоят противоположные элементы). Следовательно, на главной диагонали
стоят нули (т.к. при i=j имеем aii=-aii)

Пример.

Ясно, A'=-A

11. Эрмитова матрица: m=n и aii=-ãii (ãji - комплексно - сопряженное к aji, т.е. если A=3+2i, то
комплексно - сопряженное Ã=3-2i)
Пример

Продолжаем разговор о действиях с матрицами. А именно – в ходе изучения данной лекции


вы научитесь находить обратную матрицу. Научитесь. Даже если с математикой туго.
Что такое обратная матрица? Здесь можно провести аналогию с обратными числами:

рассмотрим, например, оптимистичное число 5 и обратное ему число . Произведение


данных чисел равно единице: . С матрицами всё похоже! Произведение матрицы
на обратную ей матрицу равно – единичной матрице, которая является
матричным аналогом числовой единицы. Однако обо всём по порядку – сначала решим
важный практический вопрос, а именно, научимся эту самую обратную матрицу находить.
Что необходимо знать и уметь для нахождения обратной матрицы? Вы должны уметь
решать определители. Вы должны понимать, что такое матрица и уметь выполнять некоторые
действия с ними.
Есть? Тогда поехали дальше. А хотя… ехать могут все, если что-то не знаете, я буду ставить
нужную ссылку по ходу объяснений.
Существует два основных метода нахождения обратной матрицы:
с помощью алгебраических дополнений и с помощью элементарных преобразований.
Сегодня мы изучим первый, более простой способ.
Начнем с самого ужасного и непонятного. Рассмотрим квадратную матрицу . Обратную
матрицу можно найти по следующей формуле:

, где – определитель матрицы , – транспонированная матрица


алгебраических дополнений соответствующих элементов матрицы .
Понятие обратной матрицы существует только для квадратных матриц, матриц «два на два», «три на
три» и т.д.

Обозначения: Как вы уже, наверное, заметили, обратная матрица обозначается надстрочным


индексом
Начнем с простейшего случая – матрицы «два на два». Чаще всего, конечно, требуется найти
обратную матрицу для матрицы «три на три», но, тем не менее, настоятельно рекомендую изучить
более простое задание, для того чтобы усвоить общий принцип решения.
Пример:

Найти обратную матрицу для матрицы


Решаем. Последовательность действий удобно разложить по пунктам.
1) Сначала находим определитель матрицы.

Если с пониманием сего действа плоховато, ознакомьтесь с материалом


Как вычислить определитель?
Важно! В том случае, если определитель матрицы равен нулю – обратной матрицы не существует.

В рассматриваемом примере, как выяснилось, , а значит, всё в порядке.


2) Находим матрицу миноров .
Для решения нашей задачи не обязательно знать, что такое минор, однако, желательно ознакомиться
со статьей Как вычислить определитель.
Матрица миноров имеет такие же размеры, как и матрица , то есть в данном случае .
Дело за малым, осталось найти четыре числа и поставить их вместо звездочек.

Возвращаемся к нашей матрице

Сначала рассмотрим левый верхний элемент:


Как найти его минор?
А делается это так: Мысленно вычеркиваем строку и столбец, в котором находится данный элемент:

Оставшееся число и является минором данного элемента, которое записываем в нашу матрицу
миноров:

Рассматриваем следующий элемент матрицы :

Мысленно вычеркиваем строку и столбец, в котором стоит данный элемент:


То, что осталось, и есть минор данного элемента, который записываем в нашу матрицу

Аналогично рассматриваем элементы второй строки и находим их миноры:

Готово.

– матрица миноров соответствующих элементов матрицы .


3) Находим матрицу алгебраических дополнений .
Это просто. В матрице миноров нужно поменять знаки у двух чисел:
Именно у этих чисел, которые я обвел в кружок!

– матрица алгебраических дополнений соответствующих элементов матрицы .


И всего-то лишь…
4) Находим транспонированную матрицу алгебраических дополнений .
Что такое транспонирование матрицы, и с чем это едят, смотрите в лекции Действия с матрицами.

– транспонированная матрица алгебраических дополнений соответствующих


элементов матрицы .
5) Ответ.

Вспоминаем нашу формулу


Всё найдено!
Таким образом, обратная матрица:

Как проверить решение? Необходимо выполнить матричное умножение либо


Проверка:

Получена уже упомянутая единичная матрица – это матрица с единицами на главной диагонали и
нулями в остальных местах.
Таким образом, обратная матрица найдена правильно.
Если провести действие , то в результате тоже получится единичная матрица. Это один из
немногих случаев, когда умножение матриц перестановочно , более подробную информацию можно
найти в статье Свойства операций над матрицами. Матричные выражения. Также заметьте, что в
ходе проверки константа (дробь) выносится вперёд и обрабатывается в самом конце – после
матричного умножения. Это стандартный приём.
Переходим к более распространенному на практике случаю – матрице «три на три»:
Пример:

Найти обратную матрицу для матрицы


Алгоритм точно такой же, как и для случая «два на два».

Обратную матрицу найдем по формуле: , где – транспонированная матрица


алгебраических дополнений соответствующих элементов матрицы .
1) Находим определитель матрицы.
Здесь определитель раскрыт по первой строке.

Также не забываем, что , а значит, всё нормально – обратная матрица существует.


2) Находим матрицу миноров .

Матрица миноров имеет размерность «три на три» , и нам нужно найти девять чисел.
Я подробно рассмотрю парочку миноров:
Рассмотрим следующий элемент матрицы:

МЫСЛЕННО вычеркиваем строку и столбец, в котором находится данный элемент:

Оставшиеся четыре числа записываем в определитель «два на два»

Этот определитель «два на два» и является минором данного элемента. Его нужно вычислить:

Всё, минор найден, записываем его в нашу матрицу миноров:

Как вы, наверное, догадались, необходимо вычислить девять определителей «два на два». Процесс,
конечно, муторный, но случай не самый тяжелый, бывает хуже.
Ну и для закрепления – нахождение еще одного минора в картинках:
Остальные миноры попробуйте вычислить самостоятельно.
Окончательный результат:

– матрица миноров соответствующих элементов матрицы .


То, что все миноры получились отрицательными – чистая случайность.
3) Находим матрицу алгебраических дополнений .
В матрице миноров необходимо сменить знаки строго у следующих элементов:

В данном случае:

– матрица алгебраических дополнений соответствующих элементов


матрицы .
4) Находим транспонированную матрицу алгебраических дополнений .

– транспонированная матрица алгебраических дополнений


соответствующих элементов матрицы .
5) Ответ:

Проверка:
Таким образом, обратная матрица найдена правильно.
Как оформить решение на чистовик? Примерный образец чистового оформления задания можно
найти на странице Правило Крамера. Метод обратной матрицы в параграфе, где идет речь о
матричном методе решения системы линейных уравнений. По существу, основная часть упомянутой
задачи – и есть поиск обратной матрицы.
Нахождение обратной матрицы для матрицы «четыре на четыре» не рассматриваем, так как такое
задание может дать только преподаватель-садист (чтобы студент вычислил один определитель
«четыре на четыре» и 16 определителей «три на три»). В моей практике встретился только один
такой случай, и заказчик контрольной работы заплатил за мои мучения довольно дорого =).
В ряде учебников, методичек можно встретить несколько другой подход к нахождению обратной
матрицы, однако я рекомендую пользоваться именно вышеизложенным алгоритмом решения.
Почему? Потому что вероятность запутаться в вычислениях и знаках – гораздо меньше.
Иногда обратную матрицу требуется найти методом Гаусса-Жордана, но второй способ доступен
для студентов с приличной техникой элементарных преобразований.

Тема 2. Элементы линейной алгебры


Системы линейных уравнений. Решение системы линейных уравнений в матричной
форме.
Цель занятия: изучить систему уравнений и научиться их решать.
Образовательная :Научиться составлять матрицу из данной системы линейных алгебраических
уравнений (СЛАУ), решать СЛАУ матричным методом.
Развивающая: Способствовать развитию внимания, логического мышления, умения анализировать
и делать выводы.
Воспитательная: Воспитывать волю и упорство для достижения конкретных результатов.
Мы рассмотрим методы решения системы линейных уравнений. В курсе высшей математики
системы линейных уравнений требуется решать как в виде отдельных заданий, например, «Решить
систему по формулам Крамера», так и в ходе решения остальных задач. С системами линейных
уравнений приходится иметь дело практически во всех разделах высшей математики.
Сначала немного теории. Что в данном случае обозначает математическое слово «линейных»? Это
значит, что в уравнения системы все переменные входят в первой степени: без всяких
причудливых вещей вроде и т.п., от которых в восторге бывают только участники
математических олимпиад.
В высшей математике для обозначения переменных используются не только знакомые с детства
буквы .
Довольно популярный вариант – переменные с индексами: .
Либо начальные буквы латинского алфавита, маленькие и большие:
Не так уж редко можно встретить греческие буквы: – известные многим «альфа, бета,
гамма». А также набор с индексами, скажем, с буквой «мю»:

Использование того или иного набора букв зависит от раздела высшей математики, в котором мы
сталкиваемся с системой линейных уравнений. Так, например, в системах линейных уравнений,
встречающихся при решении интегралов, дифференциальных уравнений традиционно принято
использовать обозначения
Но как бы ни обозначались переменные, принципы, методы и способы решения системы линейных
уравнений от этого не меняются. Таким образом, если Вам встретится что-нибудь страшное типа
, не спешите в страхе закрывать задачник, в конце концов, вместо можно
нарисовать солнце, вместо – птичку, а вместо – рожицу (преподавателя). И, как ни смешно,
систему линейных уравнений с данными обозначениями тоже можно решить.
Что-то у меня есть такое предчувствие, что статья получится довольно длинной, поэтому небольшое
оглавление. Итак, последовательный «разбор полётов» будет таким::
– Решение системы линейных уравнений методом подстановки («школьный метод»);
– Решение системы методом по членного сложения (вычитания) уравнений системы;
– Решение системы по формулам Крамера;
– Решение системы с помощью обратной матрицы;
– Решение системы методом Гаусса.
С системами линейных уравнений все знакомы из школьного курса математики. По сути дела,
начинаем с повторения.
Решение системы линейных уравнений методом подстановки
Данный метод также можно назвать «школьным методом» или методом исключения неизвестных.
Образно говоря, его еще можно назвать «недоделанным методом Гаусса».
Пример 1

Решить систему линейных уравнений:


Здесь у нас дана система из двух уравнений с двумя неизвестными. Обратите внимание, что
свободные члены (числа 5 и 7) расположены в левой части уравнения. Вообще говоря, без разницы,
где они находятся, слева или справа, просто в задачах по высшей математике нередко они
расположены именно так. И такая запись не должна приводить в замешательство, при

необходимости систему всегда можно записать «как обычно»: . Не забываем, что при
переносе слагаемого из части в часть у него нужно поменять знак.
Что значит решить систему линейных уравнений?
Решить систему уравнений – это значит найти множество её решений. Решение системы
представляет собой набор значений всех входящих в неё переменных, который обращает каждое
уравнение системы в верное равенство. Кроме того, система может быть несовместной (не иметь
решений).
Решаем: из первого уравнения выразим:
Полученное выражение подставляем во второе уравнение:

Раскрываем скобки, приводим подобные слагаемые и находим значение :

Далее вспоминаем про то, от чего плясали:


Значение нам уже известно, осталось найти:
Ответ:
После того, как решена любая система уравнений любым способом, настоятельно рекомендую
выполнить проверку (устно, на черновике либо калькуляторе). Благо, делается это легко и быстро.
1) Подставляем найденный ответ в первое уравнение :

– получено верное равенство.


2) Подставляем найденный ответ во второе уравнение :

– получено верное равенство.


Или, если говорить проще, «всё сошлось»
Рассмотренный способ решения не является единственным, из первого уравнения можно было
выразить , а не .
Можно наоборот – что-нибудь выразить из второго уравнения и подставить в первое
уравнение. Кстати, заметьте, самый невыгодный из четырех способов – выразить из второго
уравнения:

Получаются дроби, а оно зачем? Есть более рациональное решение.


Тем не менее, в ряде случаев без дробей всё-таки не обойтись. В этой связи обращаю Ваше

внимание на то, как я записал выражение. Не так: , и ни в коем случае не


так: .
Если в высшей математике Вы имеете дело с дробными числами, то все вычисления старайтесь
проводить в обыкновенных неправильных дробях.

Именно , а не или !
Запятую можно использовать лишь иногда, в частности, если – это окончательный ответ какой-
нибудь задачи, и с этим числом больше не нужно выполнять никаких действий.
Пример 2
Решить систему линейных уравнений с тремя неизвестными

Похожая система уравнений часто возникает при использовании так называемого метода
неопределенных коэффициентов, когда мы находим интеграл от дробно-рациональной функции.
Рассматриваемая система взята мной как раз оттуда.
При нахождении интеграла – цель быстро найти значения коэффициентов , а не изощряться
формулами Крамера, методом обратной матрицы и т.д. Поэтому, в данном случае уместен именно
метод подстановки.
Когда дана любая система уравнений, в первую очередь желательно выяснить, а нельзя ли ее как-
нибудь сразу упростить? Анализируя уравнения системы, замечаем, что второе уравнение системы
можно разделить на 2, что мы и делаем:

Справка: математический знак обозначает «из этого следует это», он часто используется в ходе
решения задач.
Теперь анализируем уравнения, нам нужно выразить какую-нибудь переменную через остальные.
Какое уравнение выбрать? Наверное, Вы уже догадались, что проще всего для этой цели взять
первое уравнение системы:
Здесь без разницы, какую переменную выражать, можно было с таким же успехом выразить
или
Далее, выражение для подставляем во второе и третье уравнения системы:

Раскрываем скобки и приводим подобные слагаемые:

Третье уравнение делим на 2:

Из второго уравнения выразим и подставим в третьей уравнение:

Практически всё готово, из третьего уравнения находим:


Из второго уравнения:
Из первого уравнения:
Ответ:
Проверка: Подставим найденные значения переменных в левую часть каждого уравнения системы:
1)
2)
3)
Получены соответствующие правые части уравнений, таким образом, решение найдено верно.
Пример 3
Решить систему линейных уравнений с 4 неизвестными

Это пример для самостоятельного решения (ответ в конце урока).


Решение системы методом по членного сложения (вычитания) уравнений системы
В ходе решения систем линейных уравнений нужно стараться использовать не «школьный метод», а
метод по членного сложения (вычитания) уравнений системы. Почему? Это экономит время и
упрощает вычисления, впрочем, сейчас станет всё понятнее.
Пример 4
Решить систему линейных уравнений:

Я взял ту же систему, что и первом примере.


Анализируя систему уравнений, замечаем, что коэффициенты при переменной одинаковы по
модулю и противоположны по знаку (–1 и 1). В такой ситуации уравнения можно сложить по
членно:

Действия, обведенные красным цветом, выполняются МЫСЛЕННО.


Как видите, в результате по членного сложения у нас пропала переменная . В этом, собственно, и
состоит суть метода – избавиться от одной из переменных.
Теперь всё просто: – подставляем в первое уравнение системы (можно и во
второе, но это не так выгодно – там числа больше):

В чистовом оформлении решение должно выглядеть примерно так:

Ответ:
У некоторых явно возник вопрос: «Зачем все эти изыски, если можно просто выразить одну
переменную через другую и подставить во второе уравнение?».
Пример 5
Решить систему линейных уравнений:

В данном примере можно использовать «школьный» метод, но большой минус состоит в том, что
когда мы будем выражать какую-либо переменную из любого уравнения, то получим решение в
обыкновенных дробях. А возня с дробями займет время, к тому же, если у Вас не «набита рука» на
действиях с дробями, то велика вероятность допустить ошибку.
Поэтому целесообразно использовать по членное сложение (вычитание) уравнений. Анализируем
коэффициенты при соответствующих переменных:
Как видим числа в парах (3 и 4), (4 и –3) – разные, поэтому, если сложить (вычесть) уравнения прямо
сейчас, то от переменной мы не избавимся. Таким образом, хотелось бы видеть в одной из пар
одинаковые по модулю числа, например, 20 и 20 либо 20 и –20.
Будем рассматривать коэффициенты при переменной :

Подбираем такое число, которое делилось бы и на 3 и на 4, причем оно должно быть как можно
меньше. В математике такое число называется наименьшим общим кратным. Если Вы затрудняетесь
с подбором, то можно просто перемножить коэффициенты:
Далее:

Первое уравнение умножаем на

Второе уравнение умножаем на


В результате:

Вот теперь из первого уравнения по членно вычитаем второе. На всякий случай привожу еще раз
действия, которые проводятся мысленно:

Следует отметить, что можно было бы наоборот – из второго уравнения вычесть первое, это ничего
не меняет.

Теперь подставляем найденное значение в какое-нибудь из уравнений системы, например, в


первое:

Ответ:
Решим систему другим способом. Рассмотрим коэффициенты при переменной

Очевидно, что вместо пары коэффициентов (4 и –3) нам нужно получить 12 и –12.
Для этого первое уравнение умножаем на 3, второе уравнение умножаем на 4:

По членно складываем уравнения и находим значения переменных:


Ответ:
Второй способ несколько рациональнее, чем первый, так как складывать проще и приятнее чем
вычитать.
В высшей математике всегда стремимся складывать и умножать, а не вычитать и делить.
Пример 6
Решить систему линейных уравнений:

Это пример для самостоятельного решения


Матричный метод может применяться в решении систем линейных уравнений, в которых число
неизвестных равно числу уравнений, то есть систем линейных уравнений с квадратной матрицей
коэффициентов при неизвестных.
Другое условие применимости матричного метода – не вырожденность матрицы коэффициентов при
неизвестных, то есть неравенство нулю определителя этой матрицы.
Метод Гаусса – это просто! Почему? Известный немецкий математик Иоганн Карл Фридрих Гаусс
еще при жизни получил признание величайшего математика всех времен, гения и даже прозвище
«короля математики». А всё гениальное, как известно – просто! Кстати, на деньги попадают не
только лохи, но еще и гении – портрет Гаусса красовался на купюре в 10 дойчмарок (до введения
евро), и до сих пор Гаусс загадочно улыбается немцам с обычных почтовых марок.
Метод Гаусса прост тем, что для его освоения ДОСТАТОЧНО ЗНАНИЙ
ПЯТИКЛАССНИКА. Необходимо уметь складывать и умножать! Не случайно метод
последовательного исключения неизвестных преподаватели часто рассматривают на школьных
математических факультативах. Парадокс, но у студентов метод Гаусса вызывает наибольшие
сложности. Ничего удивительного – всё дело в методике, и я постараюсь в доступной форме
рассказать об алгоритме метода.
Сначала немного систематизируем знания о системах линейных уравнений. Система линейных
уравнений может:
1) Иметь единственное решение.
2) Иметь бесконечно много решений.
3) Не иметь решений (быть несовместной).
Метод Гаусса – наиболее мощный и универсальный инструмент для нахождения
решения любой системы линейных уравнений. Как мы помним, правило Крамера и матричный
метод непригодны в тех случаях, когда система имеет бесконечно много решений или несовместна.
Вернемся к простейшей системе с урока Как решить систему линейных уравнений?

и решим ее методом Гаусса.


На первом этапе нужно записать расширенную матрицу системы:

. По какому принципу записаны коэффициенты, думаю, всем видно. Вертикальная


черта внутри матрицы не несёт никакого математического смысла – это просто отчеркивание для
удобства оформления.
Справка: рекомендую запомнить термины линейной алгебры. Матрица системы – это матрица,
составленная только из коэффициентов при неизвестных, в данном примере матрица

системы: . Расширенная матрица системы – это та же матрица системы плюс столбец

свободных членов, в данном случае: . Любую из матриц можно для краткости называть
просто матрицей.
После того, как расширенная матрица системы записана, с ней необходимо выполнить некоторые
действия, которые также называются элементарными преобразованиями.
Существуют следующие элементарные преобразования:
1) Строки матрицы можно переставлять местами. Например, в рассматриваемой матрице можно
безболезненно переставить первую и вторую строки:
2) Если в матрице есть (или появились) пропорциональные (как частный случай – одинаковые)
строки, то следует удалить из матрицы все эти строки кроме одной. Рассмотрим, например

матрицу . В данной матрице последние три строки пропорциональны, поэтому

достаточно оставить только одну из них: .


3) Если в матрице в ходе преобразований появилась нулевая строка, то ее также следует удалить.
Рисовать не буду, понятно, нулевая строка – это строка, в которой одни нули.
4) Строку матрицы можно умножить (разделить) на любое число, отличное от нуля. Рассмотрим,

например, матрицу . Здесь целесообразно первую строку разделить на –3, а вторую

строку – умножить на 2: . Данное действие очень полезно, поскольку


упрощает дальнейшие преобразования матрицы.
5) Это преобразование вызывает наибольшие затруднения, но на самом деле ничего сложного тоже
нет. К строке матрицы можно прибавить другую строку, умноженную на число, отличное от нуля.

Рассмотрим нашу матрицу из практического примера: . Сначала я распишу

преобразование очень подробно. Умножаем первую строку на –2: ,


и ко второй строке прибавляем первую строку умноженную на –

2: . Теперь первую строку можно разделить «обратно»

на –2: . Как видите, строка, которую


ПРИБАВЛЯЛИ – не изменилась. Всегда меняется строка, К КОТОРОЙ ПРИБАВЛЯЮТ.
На практике так подробно, конечно, не расписывают, а пишут короче:

Еще раз: ко второй строке прибавили первую строку, умноженную на –2. Умножают строку обычно
устно или на черновике, при этом мысленный ход расчётов примерно такой:

«Переписываю матрицу и переписываю первую строку: »


«Сначала первый столбец. Внизу мне нужно получить ноль. Поэтому единицу вверху умножаю на –
2: , и ко второй строке прибавляю первую: 2 + (–2) = 0. Записываю результат во вторую
строку: »
«Теперь второй столбец. Вверху –1 умножаю на –2: . Ко второй строке прибавляю

первую: 1 + 2 = 3. Записываю результат во вторую строку: »


«И третий столбец. Вверху –5 умножаю на –2: . Ко второй строке прибавляю первую:

–7 + 10 = 3. Записываю результат во вторую строку: »


Пожалуйста, тщательно осмыслите этот пример и разберитесь в последовательном алгоритме
вычислений, если вы это поняли, то метод Гаусса практически «в кармане». Но, конечно, над этим
преобразованием мы еще поработаем.
Элементарные преобразования не меняют решение системы уравнений: рассмотренные
манипуляции нельзя использовать, если Вам предложено задание, где матрицы даны «сами по себе».
Например, при «классических» действиях с матрицами что-то переставлять внутри матриц ни в коем
случае нельзя!

Вернемся к нашей системе . Она практически разобрана по косточкам.


Запишем расширенную матрицу системы и с помощью элементарных преобразований приведем ее
к ступенчатому виду:

(1) Ко второй строке прибавили первую строку, умноженную на –2. И снова: почему первую строку
умножаем именно на –2? Для того чтобы внизу получить ноль, а значит, избавиться от одной
переменной во второй строке.
(2) Делим вторую строку на 3.

Цель элементарных преобразований – привести матрицу к ступенчатому виду: .


В оформлении задания прямо так и отчеркивают простым карандашом «лестницу», а также обводят
кружочками числа, которые располагаются на «ступеньках». Сам термин «ступенчатый вид» не
вполне теоретический, в научной и учебной литературе он часто называется трапециевидный
вид или треугольный вид.
В результате элементарных преобразований получена эквивалентная исходной система уравнений:

Теперь систему нужно «раскрутить» в обратном направлении – снизу вверх, этот процесс
называется обратным ходом метода Гаусса.
В нижнем уравнении у нас уже готовый результат: .
Рассмотрим первое уравнение системы и подставим в него уже известное значение
«игрек»:

Ответ:
Рассмотрим наиболее распространенную ситуацию, когда методом Гаусса требуется решить систему
трёх линейных уравнений с тремя неизвестными.
Пример 1
Решить методом Гаусса систему уравнений:
Запишем расширенную матрицу системы:

Сейчас я сразу нарисую результат, к которому мы придём в ходе решения:

И повторюсь, наша цель – с помощью элементарных преобразований привести матрицу к


ступенчатому виду. С чего начать действия?
Сначала смотрим на левое верхнее число:

Почти всегда здесь должна находиться единица. Вообще говоря, устроит и –1 (а иногда и другие
числа), но как-то так традиционно сложилось, что туда обычно помещают единицу. Как
организовать единицу? Смотрим на первый столбец – готовая единица у нас есть! Преобразование
первое: меняем местами первую и третью строки:

Теперь первая строка у нас останется неизменной до конца решения. Уже легче.
Единица в левом верхнем углу организована. Теперь нужно получить нули вот на этих местах:

Нули получаем как раз с помощью «трудного» преобразования. Сначала разбираемся со второй
строкой (2, –1, 3, 13). Что нужно сделать, чтобы на первой позиции получить ноль? Нужно ко второй
строке прибавить первую строку, умноженную на –2. Мысленно или на черновике умножаем
первую строку на –2: (–2, –4, 2, –18). И последовательно проводим (опять же мысленно или на
черновике) сложение, ко второй строке прибавляем первую строку, уже умноженную на –2:

Результат записываем во вторую строку:


Аналогично разбираемся с третьей строкой (3, 2, –5, –1). Чтобы получить на первой позиции ноль,
нужно к третьей строке прибавить первую строку, умноженную на –3. Мысленно или на черновике
умножаем первую строку на –3: (–3, –6, 3, –27). И к третьей строке прибавляем первую строку,
умноженную на –3:

Результат записываем в третью строку:

На практике эти действия обычно выполняются устно и записываются в один шаг:

Не нужно считать всё сразу и одновременно. Порядок вычислений и «вписывания»


результатов последователен и обычно такой: сначала переписываем первую строку, и пыхтим себе
потихонечку – ПОСЛЕДОВАТЕЛЬНО и ВНИМАТЕЛЬНО:

А мысленный ход самих расчётов я уже рассмотрел выше.


Далее нужно получить единицу на следующей «ступеньке»:

В данном примере это сделать легко, вторую строку делим на –5 (поскольку там все числа делятся
на 5 без остатка). Заодно делим третью строку на –2, ведь чем меньше числа, тем проще решение:

На заключительном этапе элементарных преобразований нужно получить еще один ноль здесь:
Для этого к третьей строке прибавляем вторую строку, умноженную на –2:

Попробуйте разобрать это действие самостоятельно – мысленно умножьте вторую строку на –2 и


проведите сложение.
Последнее выполненное действие – причёска результата, делим третью строку на 3.
В результате элементарных преобразований получена эквивалентная исходной система линейных
уравнений:

Круто.
Теперь в действие вступает обратный ход метода Гаусса. Уравнения «раскручиваются» снизу вверх.
В третьем уравнении у нас уже готовый результат:
Смотрим на второе уравнение: . Значение «зет» уже известно, таким образом:

И, наконец, первое уравнение: . «Игрек» и «зет» известны, дело за малым:

Ответ:
Как уже неоднократно отмечалось, для любой системы уравнений можно и нужно сделать проверку
найденного решения, благо, это несложно и быстро.
Пример 2
Решить систему линейных уравнений методом Гаусса

Это пример для самостоятельного решения, образец чистового оформления и ответ в конце урока.
Следует отметить, что ваш ход решения может не совпасть с моим ходом решения, и это –
особенность метода Гаусса. Но вот ответы обязательно должны получиться одинаковыми!
Пример 3
Решить систему линейных уравнений методом Гаусса
Запишем расширенную матрицу системы и с помощью элементарных преобразований приведем ее к
ступенчатому виду:

Смотрим на левую верхнюю «ступеньку». Там у нас должна быть единица. Проблема состоит в том,
что в первом столбце единиц нет вообще, поэтому перестановкой строк ничего не решить. В таких
случаях единицу нужно организовать с помощью элементарного преобразования. Обычно это можно
сделать несколькими способами. Я поступил так:
(1) К первой строке прибавляем вторую строку, умноженную на –1. То есть, мысленно умножили
вторую строку на –1 и выполнили сложение первой и второй строки, при этом вторая строка у нас не
изменилась.

Теперь слева вверху «минус один», что нас вполне устроит. Кто хочет получить +1, может
выполнить дополнительное телодвижение: умножить первую строку на –1 (сменить у неё знак).
Дальше алгоритм работает уже по накатанной колее:

(2) Ко второй строке прибавили первую строку, умноженную на 5. К третьей строке прибавили
первую строку, умноженную на 3.
(3) Первую строку умножили на –1, в принципе, это для красоты. У третьей строки также сменили
знак и переставили её на второе место, таким образом, на второй «ступеньке у нас появилась нужная
единица.
(4) К третьей строке прибавили вторую строку, умноженную на 2.
(5) Третью строку разделили на 3.
Скверным признаком, который свидетельствует об ошибке в вычислениях (реже – об опечатке),
является «плохая» нижняя строка. То есть, если бы у нас внизу получилось что-нибудь

вроде , и, соответственно, , то с большой долей вероятности


можно утверждать, что допущена ошибка в ходе элементарных преобразований.

Заряжаем обратный ход, в оформлении примеров часто не переписывают саму систему, а уравнения
«берут прямо из приведенной матрицы». Обратный ход, напоминаю, работает, снизу вверх. Да тут
подарок получился:

Ответ: .
Пример 4
Решить систему линейных уравнений методом Гаусса
Это пример для самостоятельного решения, он несколько сложнее. Ничего страшного, если кто-
нибудь запутается. Полное решение и образец оформления в конце урока. Ваше решение может
отличаться от моего решения.
В последней части рассмотрим некоторые особенности алгоритма Гаусса.
Первая особенность состоит в том, что иногда в уравнениях системы отсутствуют некоторые
переменные, например:

Как правильно записать расширенную матрицу системы? Об этом моменте я уже рассказывал на
уроке Правило Крамера. Матричный метод. В расширенной матрице системы на месте
отсутствующих переменных ставим нули:

Кстати, это довольно легкий пример, поскольку в первом столбце уже есть один ноль, и предстоит
выполнить меньше элементарных преобразований.
Вторая особенность состоит вот в чём. Во всех рассмотренных примерах на «ступеньки» мы
помещали либо –1, либо +1. Могут ли там быть другие числа? В ряде случаев могут. Рассмотрим

систему: .
Здесь на левой верхней «ступеньке» у нас двойка. Но замечаем тот факт, что все числа в первом
столбце делятся на 2 без остатка – и другая двойка и шестерка. И двойка слева вверху нас устроит!
На первом шаге нужно выполнить следующие преобразования: ко второй строке прибавить первую
строку, умноженную на –1; к третьей строке прибавить первую строку, умноженную на –3. Таким
образом, мы получим нужные нули в первом столбце.

Или еще такой условный пример: . Здесь тройка на второй «ступеньке» тоже нас
устраивает, поскольку 12 (место, где нам нужно получить ноль) делится на 3 без остатка.
Необходимо провести следующее преобразование: к третьей строке прибавить вторую строку,
умноженную на –4, в результате чего и будет получен нужный нам ноль.
Метод Гаусса универсален, но есть одно своеобразие. Уверенно научиться решать системы другими
методами (методом Крамера, матричным методом) можно буквально с первого раза – там очень
жесткий алгоритм. Но вот чтобы уверенно себя чувствовать в методе Гаусса, следует «набить руку»,
и прорешать хотя бы 5-10 систем. Поэтому поначалу возможны путаница, ошибки в вычислениях, и
в этом нет ничего необычного или трагического.
Дождливая осенняя погода за окном.... Поэтому для всех желающих более сложный пример для
самостоятельного решения:
Пример 5
Решить методом Гаусса систему четырёх линейных уравнений с четырьмя неизвестными.

Такое задание на практике встречается не так уж и редко. Думаю, даже чайнику, который
обстоятельно изучил эту страницу, интуитивно понятен алгоритм решения такой системы.
Принципиально всё так же – просто действий больше.
Случаи, когда система не имеет решений (несовместна) или имеет бесконечно много решений,
рассмотрены на уроке Несовместные системы и системы с общим решением. Там же можно
закрепить рассмотренный алгоритм метода Гаусса.
Желаю успехов!
Решения и ответы:
Пример 2: Решение: Запишем расширенную матрицу системы и с помощью элементарных
преобразований приведем ее к ступенчатому виду.

Выполненные элементарные преобразования:


(1) Ко второй строке прибавили первую строку, умноженную на –2. К третьей строке прибавили
первую строку, умноженную на –1. Внимание! Здесь может возникнуть соблазн из третьей строки
вычесть первую, крайне не рекомендую вычитать – сильно повышается риск ошибки. Только
складываем!
(2) У второй строки сменили знак (умножили на –1). Вторую и третью строки поменяли
местами. Обратите внимание, что на «ступеньках» нас устраивает не только единица, но еще и –1,
что даже удобнее.
(3) К третьей строке прибавили вторую строку, умноженную на 5.
(4) У второй строки сменили знак (умножили на –1). Третью строку разделили на 14.
Обратный ход:

Ответ: .
Пример 4: Решение: Запишем расширенную матрицу системы и с помощью элементарных
преобразований приведем ее к ступенчатому виду:

Выполненные преобразования:
(1) К первой строке прибавили вторую. Таким образом, организована нужная единица на левой
верхней «ступеньке».
(2) Ко второй строке прибавили первую строку, умноженную на 7. К третьей строке прибавили
первую строку, умноженную на 6.
Со второй «ступенькой» всё хуже, «кандидаты» на неё – числа 17 и 23, а нам нужна либо единичка,
либо –1. Преобразования (3) и (4) будут направлены на получение нужной единицы

(3) К третьей строке прибавили вторую, умноженную на –1.


(4) Ко второй строке прибавили третью, умноженную на –3.
Нужная вещь на второй ступеньке получена.
(5) К третьей строке прибавили вторую, умноженную на 6.
(6) Вторую строку умножили на –1, третью строку разделили на -83.
Обратный ход:
Ответ:
Пример 5: Решение: Запишем матрицу системы и с помощью элементарных преобразований
приведем ее к ступенчатому виду:

Выполненные преобразования:
(1) Первую и вторую строки поменяли местами.
(2) Ко второй строке прибавили первую строку, умноженную на –2. К третьей строке прибавили
первую строку, умноженную на –2. К четвертой строке прибавили первую строку, умноженную на –
3.
(3) К третьей строке прибавили вторую, умноженную на 4. К четвертой строке прибавили вторую,
умноженную на –1.
(4) У второй строки сменили знак. Четвертую строку разделили на 3 и поместили вместо третьей
строки.
(5) К четвертой строке прибавили третью строку, умноженную на –5.
Обратный ход:

Ответ:

ТЕМА. Множества действительных чисел. Комплексные числа.


Цели урока:
Обучающая: познакомить с понятиями «множество», «элемент множества», «подмножество»;
научить определять число элементов множества; учить определять принадлежность элементов
множеству и его подмножеству.
Развивающая: развивать логическое мышление, внимания, воображение, умение анализировать,
сравнивать, обобщать.
Воспитывающая: воспитывать интерес у учащихся к предмету, коммуникативные навыки.

Первые числа, которыми люди начали пользоваться в доисторические ещё времена — это
натуральные числа, то есть целые и положительные: 1, 2, 3, . . .
Натуральные числа — это числа, применяемые для счёта предметов. Натуральные числа можно
использовать в качестве номеров.
Наименьшее натуральное число — единица¹. Числа 21, 249, 30988 являются натуральными. Все
вместе они составляют множество натуральных чисел, обозначаемое буквой N:
N = {1, 2, 3, . . .}.
Что же такое множество? Это одно из первичных понятий математики, т. е. таких, которые лежат в
основе логической системы и уже не определяются через другие понятия. Интуитивно мы понимаем,
что множество — это набор или совокупность элементов, объединенных каким-либо общим
признаком.
Множества обычно обозначаются заглавными буквами. Множество натуральных чисел мы можем
условно изобразить вот так: N = {1, 2, 3, . . .}.

Но числа бывают не только натуральными. Индийцы изобрели число ноль и отрицательные числа.
Теперь они для нас привычны, но когда-то европейцы — древние греки и римляне — долгое время
обходились без нуля. Сейчас нам трудно это представить.
Натуральные числа, целые отрицательные числа и ноль вместе составляют множество целых чисел,
которое обозначается Z :
Z = {0, ±1, ±2, ±3, . . .}
Например, получая в тригонометрическом уравнении серию решений, мы пишем: n ∈ Z, и это
означает, что n — целое число.
Очевидно, множество целых чисел включает в себя множество натуральных:

Кроме целых чисел, однако, имеются ещё и дроби. Напомним, что дробь — это часть, доля,

выражение вида (где p — целое, а q — натуральное). Например, — это «одна часть из трёх»,

0,25 — это двадцать пять сотых. Десятичные дроби также можно записать в виде .

Например, .

Целые числа (положительные и отрицательные) также можно записать в виде . Например, в виде
дроби со знаменателем 1:

Стало быть, целые числа — частный случай дробей.

Числа, которые можно записать в виде дроби , называются рациональными. Множество


рациональных чисел обозначается Q. Ясно, что оно включает в себя множество целых чисел.

Долгое время — в античности — считалось, что любое число можно записать в виде дроби с
числителем и знаменателем. Дело в том, что для древних греков числа и их соотношения были почти
священны. Пифагорейцы говорили: «Числа правят миром». Они верили, что все основные принципы
мироздания можно выразить языком математики, что соотношения чисел выражают гармонию,
закон и порядок природы, перед которым склоняют голову даже олимпийские боги. Греческое
искусство, особенно архитектура, подчинялось правилам, канонам. Греки точно установили, какими
должны быть пропорции в архитектуре — например, отношение диаметра колонны к её длине —
чтобы здание было гармоничным. И все эти пропорции были отношениями целых чисел.
Однако в стройной и гармоничной системе божественных пропорций наметилась досадная брешь.
Оказалось, что отношение диагонали квадрата к его стороне не выражается отношением целых
чисел! Другими словами, если мы нарисуем квадрат со стороной 1, его диагональ не выражается

никакой дробью вида .


По теореме Пифагора диагональ такого квадрата равна — то есть положительному числу,
квадрат которого равен двум. Можно доказать, что это число не является рациональным. Но сами

пифагорейцы не сразу смоги смириться с тем, что невозможно записать в виде — ведь это
наносило удар всей их философской системе!
Открытие долго держалось в тайне, пока наконец ученик Пифагора Гиппас не разгласил его. За это
Гиппас был изгнан из школы Пифагора и вскоре погиб во время кораблекрушения, в чём
современники увидели несомненное возмездие богов. А числа, которые невозможно записать в

виде , такие, как , назвали иррациональными, то есть не- разумными, неправильными.


Но иррациональные числа ничуть не хуже рациональных! Они отнюдь не ограничиваются
выражениями вида или . К ним относятся также:
число 𝜋 — отношение длины окружности к её диаметру;
число e, названное в честь Эйлера (об этом числе мы ещё расскажем);
задающее золотое сечение число φ — удивительное число Фибоначчи, вокруг которого построен
весь детективный сюжет фильма «Код да Винчи»;
числа вида
необозримое количество других чисел.
Ещё раз повторим, в чём разница между рациональными и иррациональными числами.

Рациональное число можно представить в виде дроби — например, , . А если мы просто


поделим в столбик 7 на 11, мы обнаружим интересную закономерность:
7 : 11 = 0,636363636363...
Мы видим, что цифры повторяются, то есть дробь является периодической. Таким образом, любое
рациональное число можно записать десятичной дробью — конечной или бесконечной
периодической.
А вот в числе 𝜋 = 3,1415926... цифры не заканчиваются, и никакой периодичности их следования не
наблюдается. Иррациональные числа — это бесконечные непериодические дроби.
Вместе оба множества — рациональных и иррациональных чисел — образуют множество
действительных (или вещественных) чисел, которое обозначается R (от слова real).

Возникает вопрос: это всё? Все ли числа, какие только могут быть, содержатся в множестве
действительных чисел? Или за его пределами ещё что-то есть?
Для успешной сдачи ЕГЭ других чисел не нужно. Да и, казалось бы, мы назвали все возможные
числа. Но вот какой парадокс: положительные и отрицательные числа симметрично расположены на
числовой прямой, верно? И при этом из положительных чисел можно извлечь квадратный корень, а
из отрицательных — нельзя! Не существует действительного числа, которое при возведении в
квадрат даёт −1.
Оказывается, однако, что существует числовое множество, содержащее в себе множество R и
бесконечное множество других чисел, не являющихся действительными. В этом множестве
находится мнимая единица i, для которой верно i² = −1. И называется оно множеством комплексных
чисел.
Комплексные числа служат естественным языком описания многих физических явлений. Те из вас,
кто выбрал инженерную специальность (в особенности связанную с распространением волн,
электротехникой и радиофизикой), непременно встретятся с ними. В отличие от действительных
(«вещественных») чисел, применяемых для описания материального, плотного мира «вещей»,
комплексные числа оказываются удобным инструментом для построения математических моделей
волн и колебаний всевозможной природы.
Ну а будущим физикам наверняка интересно будет узнать, что элементарные частицы живут и
взаимодействуют по законам именно комплексных чисел. Наукой, описывающей комплексный
микромир, является квантовая физика.
Действия над комплексными числами в алгебраической форме
Сложение
Пусть , и .
Тогда получается простым приведением подобных:

Вычитание
Пусть , и .
Тогда получается аналогично со сложением:

Умножение
Пусть , и .
Тогда .
Что делать на этом шаге? Все довольно просто, как Вы наверно и подумали, надо всего лишь
раскрыть скобки и привести подобные:

Определение комплексно сопряженного числа


Пусть , и .
называют комплексно сопряженным к , если и , т.е.
и .
И при перемножении
Это потребуется для нашего следующего действия.
Деление
Пусть , и .
Тогда
На этом шаге обычно все и остановилось бы, но мы сможем еще упростить выражение благодаря
знанию комплексно сопряженных чисел. Умножим числитель и знаменатель на комплексно
сопряженное число к знаменателю, получим:

Действия над комплексными числами в тригонометрической форме


Умножение
Произведением двух комплексных чисел и
будет комплексное число вида
Деление
Частным двух комплексных чисел и будет
комплексное число вида
Возведение в степень

Извлечение корня
,
Тема:Производная, правила и формулы дифференцирования
Цели урока:
Обучающая:
Применение правил и формул дифференцирования.
Развивающая:
В процессе выполнения упражнений необходимо систематически обращать внимание студентов на
приёмы вычислений, развивать аккуратность, правильно оформлять записи на доске и в тетрадях,
правильное использовать символику и терминологию.
Воспитательная:
Прививать интерес к математике, используя исторические материалы.

Пусть задана функция f(x) на интервале (a,b). Зафиксируем точку x внутри (a,b) и придадим x
приращение ∆x, MP секущая, приращение функции ∆y = f(x+∆x)-f(x). Рассмотрим отношение

это тангенс угла наклона


секущей MP, он зависит от ∆x.
Определение. Производной называется предел
отношения приращения функции к приращению
аргумента , когда приращение аргумента стремится к
нулю:

Существует несколько способов обозначения


производной, самые важные это .
Пример нахождения , используя определение:

Геометрический смысл производной

По определению устремим точку M к точке P , это эквивалентно


стремлению .
Предельное положение секущей MP это касательная к кривой в точке M , ее угловой коэффициент

равен
Следовательно, производная в точке х равна тангенсу угла наклона касательной в этой точке.
Уравнение касательной в точке имеет вид , т.к.
, то уравнение касательной примет вид . Найдем уравнение
нормали, перпендикулярной данной касательной и проходящей через точку . Из условия

перпендикулярности прямых угловой коэффициент нормали равен , а


уравнение нормали в точке примет вид

Механический смысл производной


Пусть прямолинейное движение материальной точки задано законом S = S(t). Путь, который

проследует точка за время ∆ равен ∆S = S(t+∆t)-S( t). Средняя скорость есть , мгновенная

скорость
Пример.
Пусть дан закон движения материальной точки , найти скорость точки через t = 3 сек.

Дифференциал функции
Пусть задана y = f(x) на интервале (a,b). Функция y = f(x) называется дифференцируемой в точке x,
если ∆y можно представить с помощью следующего выражения:
∆y = A∆x + α(∆)∆x
где А= const при фиксированном х и при
Теорема. Для дифференцируемости функции в точке х необходимо и достаточно, чтобы функция
имела в этой точке конечную производную.
Дифференциалом функции y = f(x) называется выражение вида dy=A - это главная линейная
часть приращения ∆y , на основании предыдущей теоремы , обозначив дифференциал
независимой переменной через dx=∆x, получим выражение для дифференциала:

Геометрический смысл дифференциала виден из следующего рисунка

, т.е. дифференциал функции равен отрезку PQ это


приращение ординаты касательной, а приращение ∆y это отрезок
Формулы дифференцирования
Таблица производных
Производная сложной функции
Если y = f(x) и u = u(x), то есть y=f[u(x)] сложная функция, где f(u) и u(x) имеют производные, то

это правило дифференцирования сложной функции.


Пример.

Производная обратной функции


Пусть задана y = f(x), тогда определена обратная функция x = ϕ(y). Для y = 5x обратная

функция , для обратная функция . Пусть y = f(x) возрастает или убывает на


(a,b) и непрерывна, тогда существует обратная функция x = ϕ(y) и ее производная

.
Примеры. Найти производную обратной тригонометрической функции y = arcsinx. Обратная
функция x = siny и , по формуле для обратной

функции .
Найдем для y = arctgx Обратная функция x =

tgy .
Логарифмическое дифференцирование
Пусть имеется функция найдем ее производную. Сначала прологарифмируем данное

выражение, получим lny = v(x)lnu(x). Теперь продифференцируем

Пример. Найти функции


Логарифмируем данную функцию lny = xlnx, теперь дифференцируем

;
Применение дифференциалов в приближенных вычислениях

Вычислить
Используем приближенное равенство
Тема :Неопределенный интеграл. Основные свойства неопределенного
интеграла. Интегрирование подстановкой и по частям. Определенный интеграл.
Вычисление определенных интегралов. Вычисление площадей плоских фигур.

Цель урока: обобщение и систематизация знаний по теме


Задачи урока:
образовательные:
углубление понимания сущности определенного интеграла путем применения его для получения
новых знаний;
развитие умений и навыков применять определенный интеграл при решении задач;
воспитательные:
воспитание познавательного интереса к учебному предмету;
воспитание у учащихся культуры мышления;
формирование умений осуществлять самоконтроль;
развивающие:
формирование умений строить доказательства, логическую цепочку рассуждений;
формирование умений проводить обобщение, переносить знания в новую ситуацию

Совокупность всех первообразных к функции называется неопределенным интегралом от


функции

Функция называется подынтегральной функцией, dx переменной интегрирования.


Другими словами, неопределенный интеграл – это общая первообразная, содержащая произвольную
постоянную, при каждом численном значении которой получается частная первообразная.
Восстановление функции по ее производной (отыскание неопределенного интеграла)
называется интегрированием этой функции. Интегрирование – операция обратная
дифференцированию.
Из определения неопределенного интеграла вытекают его свойства.
1)Производная неопределенного интеграла равна подынтегральной функции; дифференциал от
неопределенного интеграла равен подынтегральному выражению, т.е.

2)Неопределенный интеграл от дифференциала некоторой функции равен сумме этой функции и


произвольной постоянной, т.е.

3)Постоянный множитель можно вынести из-под знака интеграла, т.е. если то

4)Неопределенный интеграл от алгебраической суммы двух функций равен алгебраической сумме


интегралов от этих функций в отдельности, т.е.

19.3. Таблица основных интегралов


Приведем таблицу основных интегралов. Часть формул следует из определения интегрирования как
операции, обратной дифференцированию, и таблицы производных. В таблице C= const
произвольная постоянная, – функция от переменной число.
Таблица 6
Неопреде Неопреде
Первооб Первообраз
ленный ленный
разная ная
интеграл интеграл

Неопреде Неопреде
Первооб Первообраз
ленный ленный
разная ная
интеграл интеграл

Рассмотрим основные методы интегрирования.


Метод непосредственного интегрирования основан на непосредственном использовании формул
таблицы интегралов и свойств неопределенного интеграла.
Заметим, что дифференцирование является мощным инструментом проверки результатов
интегрирования.
Рассмотрим пример. Требуется найти значение интеграла

На основе известной формулы дифференцирования


можно сделать вывод, что искомый интеграл равен где некоторое постоянное число.
С другой стороны,

Окончательно можно сделать вывод:

В отличие от дифференцирования, где для нахождения производной использовались четкие приемы


и методы, правила нахождения производной, определение производной, для интегрирования такие
методы недоступны. При нахождении первообразной следует в основном опираться на знания
таблиц производных и первообразных.
Метод непосредственного интегрирования применим только для весьма ограниченных классов
функций. Поэтому в большинстве случаев применяются способы, описанные ниже.
Интегрирование методом замены переменной
Во многих случаях введение новой переменной интегрирования позволяет свести нахождение
данного интеграла к нахождению табличного интеграла, т.е. перейти к непосредственному
интегрированию. Такой метод называют методом подстановки или методом замены переменной.
Данный метод основан на теореме: пусть функция определена и дифференцируема на
некотором промежутке и пусть множество значений этой функции, на котором определена
функция . Тогда на множестве функция имеет первообразную, на множестве
справедлива формула

Пусть первообразная для на множестве Рассмотрим на множестве сложную


функцию По правилу дифференцирования сложной функции, учитывая, что

получаем

т.е. функция имеет на множестве первообразную , следовательно

Замечая, что .
Рассмотрим пример. Найти неопределенный интеграл

Сделаем замену Получим

Рассмотрим еще один пример. Найти интеграл

Сделаем замену Получим

Пример. Найти интеграл

Сделаем замену Получим

Рассмотрим пример. Найти интеграл и проверить ответ дифференцированием


Проверим

Рассмотрим пример. Найти интеграл и проверить ответ дифференцированием

Проверим

Рассмотрим пример. Найти интеграл

Рассмотрим пример. Найти интеграл


Контрольные вопросы
Какая функция называется первообразной?
Что представляет собой неопределенный интеграл?
Какими свойствами обладает неопределенный интеграл?
Какова идея метода замены переменной?

Определенный интеграл. Вычисление определенных интегралов. Вычисление площадей плоских


фигур. Формула Ньютона-Лейбница.
Рассмотрим непрерывную функцию y = f ( x ), заданную на отрезке [ a, b ] и сохраняющую на этом
отрезке свой знак ( рис.8 ).
Фигура, ограниченная графиком этой функции, отрезком [ a, b ] и прямыми x = a и x = b,
называется криволинейной трапецией.
Для вычисления площадей криволинейных трапеций используется следующая теорема:

Если f – непрерывная, неотрицательная функция на отрезке [a, b], и F – её первообразная на этом


отрезке, то площадь соответствующей криволинейной трапеции равна приращению первообразной
на отрезке [a, b], т.e.

Рассмотрим функцию S ( x ), заданную на отрезке [ a, b ]. Если a<x b, то S ( x ) – площадь части


криволинейной трапеции, лежащей слева от вертикальной прямой, проходящей через точку ( x, 0 ).
Отметим, что если x = a , то S ( a ) = 0, а S ( b ) = S ( S – площадь всей криволинейной трапеции).
Можно доказать, что

т.e. S ( x ) – первообразная для f ( x ). Отсюда, согласно основному свойству первообразных, для


всех x [ a, b ] имеем:
S(x)=F(x)+C,

где C – некоторая постоянная, F – одна из первообразных функции f .


Чтобы найти C , подставим x = a :

F ( a ) + C = S ( a ) = 0,

отсюда, C = -F ( a ) и S ( x ) = F ( x ) - F ( a ). Так как площадь криволинейной трапеции


равна S ( b ) , то подставляя x = b , получим:

S = S ( b ) = F ( b ) - F ( a ).

П р и м е р . Найти площадь фигуры, ограниченной кривой y = x2 и прямыми


y = 0, x = 1, x = 2 ( рис.9 ) .

Определённый интеграл. Рассмотрим другой способ вычисления площади криволинейной трапеции.


Разделим отрезок [ a, b ] на n отрезков равной длины точками:

x0 = a < x1 < x2 < x3 < …< x n - 1 < xn = b

и пусть = ( b – a ) / n = xk - xk - 1 , где k = 1, 2, …, n – 1, n .
В каждом из отрезков [ xk- 1 , xk ] как на основании построим прямоугольник высотой f ( xk - 1 ).
Площадь этого прямоугольника равна:

Ввиду непрерывности функции f (x) объединение построенных прямоугольников при


большом n ( т.e. при малом "почти совпадает" с нашей криволинейной трапецией ).
Поэтому, Sn S при больших значениях n . Это значит, что Sn S при n . Этот предел
называется интегралом функции f ( x ) от a до b или определённым интегралом :
Числа a и b называются пределами интегрирования, f ( x ) dx – подынтегральным выражением.
Итак, если f ( x ) 0 на отрезке [ a, b ], то площадь S соответствующей криволинейной трапеции
вычисляется по формуле:

Пусть функция y = f(x) непрерывна на отрезке [a; b] и F(x) - одна из первообразных функции на этом

отрезке, тогда справедлива формула Ньютона-Лейбница: .


Формулу Ньютона-Лейбница называют основной формулой интегрального исчисления.
Для доказательства формулы Ньютона-Лейбница нам потребуется понятие интеграла с переменным
верхним пределом.

Если функция y = f(x) непрерывна на отрезке [a; b], то для аргумента интеграл

вида является функцией верхнего предела. Обозначим эту функцию ,

причем эта функция непрерывная и справедливо равенство .

Действительно, запишем приращение функции , соответствующее приращению аргумента


и воспользуемся пятым свойством определенного интеграла и следствием из десятого свойства:

где .

Перепишем это равенство в виде . Если вспомнить определение

производной функции и перейти к пределу при , то получим . То есть, -


это одна из первообразных функции y = f(x) на отрезке [a; b]. Таким образом, множество всех

первообразных F(x) можно записать как , где С – произвольная


постоянная.
Вычислим F(a), используя первое свойство определенного

интеграла: , следовательно, .
Воспользуемся этим результатом при

вычислении F(b): , то

есть . Это равенство дает доказываемую формулу Ньютона-


Лейбница .

Приращение функции принято обозначать как . Пользуясь этим

обозначением, формула Ньютона-Лейбница примет вид .


Для применения формулы Ньютона-Лейбница нам достаточно знать одну из
первообразных y=F(x) подынтегральной функции y=f(x) на отрезке [a; b] и вычислить приращение
этой первообразной на этом отрезке. В статье методы интегрирования разобраны основные способы
нахождения первообразной. Приведем несколько примеров вычисления определенных интегралов
по формуле Ньютона-Лейбница для разъяснения.

Пример.

Вычислить значение определенного интеграла по формуле Ньютона-Лейбница.


Решение.

Для начала отметим, что подынтегральная функция непрерывна на отрезке [1;3],


следовательно, интегрируема на нем. (Об интегрируемых функциях мы говорили в разделе функции,
для которых существует определенный интеграл).

Из таблицы неопределенных интегралов видно, что для функции множество первообразных


для всех действительных значений аргумента (следовательно, и для ) записывается

как . Возьмем первообразную при C = 0: .


Теперь осталось воспользоваться формулой Ньютона-Лейбница для вычисления определенного

интеграла: .
Пример.

По формуле Ньютона-Лейбница вычислите определенный интеграл .


Решение.
Подынтегральная функция непрерывна на отрезке [-1;2], поэтому, интегрируема на нем.

Найдем неопределенный интеграл методом подведения под знак

дифференциала: . Так мы получили множество всех

первообразных функции для всех действительных x, следовательно, и для .


Возьмем первообразную при С=0 и применим формулу Ньютона-Лейбница:
Пример.

Вычислить определенные интегралы .


Решение.

На отрезке подынтегральная функция непрерывна, следовательно, интегрируема.

Найдем множество первообразных функции

: .

Возьмем первообразную и по формуле Ньютона-Лейбница вычислим требуемый


определенный интеграл:

Переходим ко второму определенному интегралу.

На отрезке [-1;1] подынтегральная функция не ограничена, так как , то есть, не


выполняется необходимое условие интегрируемости функции на отрезке. Более

того, не является первообразной функции на отрезке [-1;1],


поскольку точка 0, принадлежащая отрезку, не входит в область определения функции.
Следовательно, не существует определенный интеграл Римана и Ньютона-Лейбница для

функции на отрезке [-1;1].


Итак, перед применением формулы Ньютона-Лейбница обязательно нужно убедиться, что
указанный определенный интеграл существует.
Замена переменной в определенном интеграле.

Пусть функция y = f(x) определена и непрерывна на отрезке [a; b]. Множество [a; b] является
областью значений некоторой функции x = g(z), которая определена на интервале и имеет на

нем непрерывную производную, причем и ,

тогда .
Этой формулой удобно пользоваться в тех случаях, когда нам требуется вычислить

интеграл , причем неопределенный интеграл мы бы искали методом


подстановки.
Разберем на примере для ясности.
Пример.

Вычислить значение определенного интеграла .


Решение.
Подынтегральная функция непрерывна на отрезке интегрирования, следовательно, определенный
интеграл существует.

Обозначим . При x=9 имеем , а

при x=18 имеем , то есть, .

Подставляем полученные результаты в формулу :

Из таблицы неопределенных интегралов видно, что одной из первообразных функции

является функция , поэтому, по формуле Ньютона-Лейбница имеем

Можно было обойтись и без формулы .

Если методом замены переменной взять неопределенный интеграл , то мы придем к

результату .
Таким образом, по формуле Ньютона-Лейбница вычисляем определенный интеграл:
Интегрирование по частям при вычислении определенного интеграла.
Пусть на отрезке [a; b] определены и непрерывны функции u(x) и v(x) вместе со своими
производными первого порядка и функция – интегрируема, тогда на этом отрезке
интегрируема функция и справедливо

равенство .
Этой формулой удобно пользоваться в тех случаях, когда нам требуется вычислить

интеграл , причем неопределенный интеграл мы бы искали интегрированием по


частям.
Пример.

Вычислить определенный интеграл .


Решение.

Функция является интегрируемой на отрезке в силу своей


непрерывности.

Пусть u(x) = x, а , тогда ,

а . По формуле получаем

Этот пример можно решить и по-другому.

Находим множество первообразных функции интегрированием по частям и


применяем формулу Ньютона-Лейбница:
МИНИСТЕРСТВО ОБРАЗОВАНИЯ И НАУКИ КЫРГЫЗСКОЙ РЕСПУБЛИКИ
КЫРГЫЗСКИЙ НАЦИОНАЛЬНЫЙ УНИВЕРСИТЕТ
имени Ж. БАЛАСАГЫНА
ПРОФЕССИОНАЛЬНЫЙ КОЛЛЕДЖ ОТДЕЛЕНИЕ ПЕРЕВОДЧЕСКОЕ ДЕЛО

МАТЕРИАЛЫ ПРАКТЧЕСКИХ ЗАНЯТИЙ

ДИСЦИПЛИНЫ

Профессиональная математика
Специальность « Переводческое дело» , «Туризм»

Практическая работа

Тема: Вычисление обратных матриц второго и третьего порядков


Цель: закрепить навыки вычисления обратных матриц второго и третьего порядка
Теоретическая часть:
О Минором Mij элемента aij определителя D=, где i и j меняются от 1 до n, называется такой
новый определитель, который получается из данного определителя вычеркиванием строки и
столбца, содержащих данный элемент.

Например, минор M12 , соответствующий элементу a12 определителя D = | |


a11 a12
a21 a22
получается,

если вычеркнуть из определителя D первую строку и второй столбец, т.е. M12 = a 21


О Алгебраическим дополнением элемента aij определителя D называется минор Mij этого
элемента, взятый со знаком (-1)i+j
Т.о. Aij = (-1)i+j ∙ Mij

| |
1 12
Пример. Найти алгебраические дополнения элементов a13, a21, a31 определителя D= 2 2−2
−1 1−2
Решение
A13 = (-1)1+3∙M13 =(2∙1-2∙(−1)¿ = 4
A21 = (-1)2+1∙M21 = (-1) ∙(1∙(-2)–2∙(-1)) = –4
A31 = (-1)3+1 ∙ M31 = = –6
О Квадратная матрица A называется вырожденной, если ее определитель равен 0 и
невырожденной, если ее определитель отличен от нуля.
О Если A – квадратная матрица, то обратной по отношению к A называется матрица, которая
будучи умноженной на A (как справа, так и слева), дает единичную матрицу.
A-1 – обратная матрица
A-1∙A=A∙A-1=E
При условии D=≠0 обратная матрица находится по формуле
A-1 =
Вычисление обратных матриц второго и третьего порядка
Для нахождения обратной матрицы используют следующую схему:
Находят определитель матрицы A.
Находят алгебраические дополнения всех элементов aij матрицы A и записывают новую
матрицу.
Меняют местами столбцы полученной матрицы (транспонируют матрицу)
Умножают полученную матрицу на

Рассмотрим квадратную матрицу 2-го порядка:

Определение. Определителем 2-го порядка, соответствующим матрице А, называется число

Числа а11, а12, а21, а22 называются элементами определителя (они же элементы
матрицы А).
Элементы а11, а22 составляют главную диагональ, а элементы а21, а12 – побочную
диагональ.

Пусть дана квадратная матрица 3-го порядка:

.
Определение. Определителем 3-го порядка, соответствующим матрице А, называется число
D, которое определяется выражением:

Элементы а11, а22, а33 – расположены на главной диагонали, элементы а13, а22, а31 –
на побочной диагонали.

Вычисление определителей 2-го и 3-го порядка

Определитель 2-го порядка вычисляется по определению:


Пример 1

Для вычисления определителя 3-го порядка можно воспользоваться следующими


правилами:

Правило Саррюса: дописать справа к элементам определителя сначала 1-й столбец, затем 2-
й (можно внизу дописать первую и вторую строки), (рис.1), произведение элементов,
стоящих на главной диагонали определителя, а также произведения элементов, стоящих на
двух параллелях к ней, содержащих по 3 элемента – нужно взять со знаком «плюс», а
произведение элементов побочной диагонали и двух параллелях к ней, содержащих по 3
элемента – нужно взять со знаком «минус» (рис. 1). Сумма этих шести произведений дает
определитель 3-го порядка, соответствующий матрице А.

,
Рис. 1
Пример 2

Вычислить .
Решение

,
таким образом:

Правило треугольника:одно из трех слагаемых, со знаком «плюс» есть произведение


элементов главной диагонали определителя, каждое из двух других – произведение
элементов, лежащих на параллели к этой диагонали, и элемента из противоположного угла
определителя, слагаемые со знаком «минус» строятся так же, но относительно побочной
диагонали (рис.2).

(+) (-)
Рис. 2
Пример 3

Вычислить определитель по правилу треугольника: .


Решение

Свойства определителей

Рассмотрим свойства определителей на примере определителя 3-го порядка.


Рассмотрим определитель:

Определение. Минором некоторого элемента определителя называется определитель,


полученный из данного путем вычеркивания строки и столбца, на пересечении которых
стоит этот элемент. Обозначение минора .
Пример 4

Минор элемента а12: .

Определение. Алгебраическим дополнениемлюбого элемента определителя называется


минор этого элемента, взятый со своим знаком, если сумма номеров строки и столбца, на
пересечении которых стоит этот элемент, есть число четное, либо с противоположным
знаком, если эта сумма есть число нечетное. Обозначение алгебраического дополнения Аij.
Пример 5
Свойство 1.Определитель равен сумме произведений элементов какого-нибудь столбца (или
строки) на их алгебраические дополнения.

Пример 6
Вычислим определитель, разложив его по элементам 1-ой строки:

.
Свойство 2. Величина определителя не изменится, если каждую его строку заменить
столбцом с тем же номером.

Свойство 3. Перестановка двух столбцов или двух строк определителя равносильна его
умножению на (–1).

Свойство 4.Общий множитель всех элементов одного столбца или одной строки
определителя можно вынести за знак определителя.
Свойство 5. Если все элементы какой-либо строки или какого-либо столбца равны нулю, то
определитель равен нулю.
Свойство 6. Определитель, имеющий два одинаковых столбца или две одинаковых строки,
равен нулю.
Свойство 7. Определитель равен нулю, если элементы двух столбцов или двух строк
пропорциональны.

Свойство 8. Если каждый элемент некоторой строки (столбца) определителя представлен в


виде суммы двух слагаемых, то определитель равен сумме двух определителей, у которых
все строки (столбцы), кроме данной, прежние, а в данной строке (столбце) в первом
определителе стоят первые слагаемые, а во втором – вторые:

Свойство 9.Если к элементам некоторого столбца (или строки) определителя прибавить


соответствующие элементы другого столбца (или строки), умноженные на общий
множитель, то величина определителя не изменится.
Пример 7
Вычислим определитель:

,
при вычислении определителя первую строку умножили на 2 и сложили со второй, затем
разложили определитель по 2-й строке.

Свойство 10. Сумма произведений элементов какого-нибудь столбца (или строки) на


алгебраические дополнения элементов другого столбца (или строки) определителя равна
нулю.

Обратная матрица

Пусть дана квадратная матрица А порядка n.


Обратной матрицей по отношению к данной А называется матрица , которая будучи
умноженной, как справа, так и слева на данную матрицу, дает единичную матрицу.
По определению

А· = · А = Е.
Квадратная матрица называется неособенной или невырожденной, если определитель ее
отличен от нуля. В противном случае матрица называется особенной или вырожденной.
Всякая неособенная матрица имеет обратную матрицу, которую можно найти по формуле

где - определитель матрицы А, - союзная матрица по отношению


к данной матрице, в которой элементы каждой строки данной матрицы заменены
алгебраическими дополнениями элементов соответствующих столбцов. Например, для
квадратной матрицы 2-го порядка союзной является матрица

,
для квадратной матрицы 3-го порядка союзной является матрица

.
Пример

для матрицы найти обратную.


Решение
Обратную матрицу находим по формуле

Определитель матрицы , следовательно, матрица


неособенная и обратная матрица существует. Найдем алгебраические дополнения элементов
матрицы:

.
Тогда обратная матрица имеет вид
Тема : Решение системы линейных уравнений в матричной форме.

Пример.
С помощью обратной матрицы найдите решение системы линейных

уравнений .
Решение.
В матричной форме исходная система запишется как ,

где . Вычислим определитель основной матрицы и


убедимся, что он отличен от нуля. В противном случае мы не сможем решить систему

матричным методом. Имеем ,

следовательно, для матрицы А может быть найдена обратная матрица . Таким образом,
если мы отыщем обратную матрицу, то искомое решение СЛАУ определим

как . Итак, задача свелась к построению обратной матрицы . Найдем ее.

Мы знаем, что для матрицы обратная матрица может быть найдена


как , где - алгебраические дополнения
элементов .
В нашем случае

Тогда

Выполним проверку полученного решения , подставив его в матричную


форму исходной системы уравнений . Это равенство должно обратиться в
тождество, в противном случае где-то была допущена ошибка.

Следовательно, решение найдено верно.


Ответ:
или в другой записи .
Пример.

Решите СЛАУ матричным методом.


Решение.
Первое уравнение системы не содержит неизвестной переменной x2, второе – x1, третье
– x3. То есть, коэффициенты перед этими неизвестными переменными равны нулю.

Перепишем систему уравнений как . От такого вида проще

перейти к матричной форме записи СЛАУ .


Убедимся в том, что эта система уравнений может быть решена с помощью обратной
матрицы. Другими словами, покажем что :

Построим обратную матрицу с помощью матрицы из алгебраических дополнений:


тогда,

Осталось найти решение СЛАУ:


Рекомендуем выполнить проверку.
Ответ:

.
При переходе от обычного вида системы линейных алгебраических уравнений к ее
матричной форме следует быть внимательным с порядком следования неизвестных

переменных в уравнениях системы. К примеру, СЛАУ НЕЛЬЗЯ записать

как . Нужно сначала упорядочить все неизвестные переменные во


всех уравнениях системы, а потом переходить к матричной записи:

или

Также будьте внимательны с обозначением неизвестных переменных, вместо x1, x2, …,

xn могут быть любые другие буквы. Например, СЛАУ в матричной


форме запишется как .
Разберем пример.
Пример.

Найдите решение системы линейных алгебраических уравнений с


помощью обратной матрицы.
Решение.
Упорядочив неизвестные переменные в уравнениях системы, запишем ее в матичной

форме . Вычислим
определитель основной матрицы:

Он отличен от нуля, поэтому решение системы уравнений может быть найдено с помощью

обратной матрицы как . Найдем обратную матрицу по

формуле :
Получим искомое решение:

Ответ:
x = 0, y = -2, z = 3.

Пример. Найдите решение системы линейных алгебраических

уравнений матричным методом.

Решение.
Определитель основной матрицы системы равен нулю

поэтому, мы не можем применить матричный метод.


Нахождение решения подобных систем описано в разделе решение систем линейных
алгебраических уравнений.
Пример.

Решите СЛАУ матричным методом, - некоторое действительное


число.
Решение.

Система уравнений в матричной форме имеет вид . Вычислим


определитель основной матрицы системы и убедимся в том, что он отличен от нуля:

Квадратных трехчлен не обращается в ноль ни при каких действительных


значениях , так как его дискриминант отрицателен , поэтому

определитель основной матрицы системы не равен нулю ни при каких действительных .

По матричному методу имеем . Построим обратную матрицу

по формуле :

Тогда

Рекомендуем выполнить проверку полученного результата.


Ответ:

.
Тема : Решение линейных уравнений по формулам Крамера . Решение
линейных уравнений методом Гаусса.

Метод Крамера.

Метод Крамера применяется для решения систем линейных алгебраических уравнений (СЛАУ).

Формулы на примере системы из двух уравнений с двумя переменными.


Дано: Решить методом Крамера систему

относительно переменных х и у.
Решение:
Найдем определитель матрицы, составленный из коэффициентов системы
Вычисление определителей.:

Заменим в этом определителе первый столбец столбцом коэффициентов из правой части системы
и найдем его значение:

Сделаем аналогичное действие, заменив в первом определителе второй столбец:

Применим формулы Крамера и найдем значения переменных:

и .

Пример 1: Решить систему уравнений:

относительно переменных х и у.
Решение:
Найдем определитель матрицы, составленный из коэффициентов системы :
Заменим в этом определителе первый столбец столбцом коэффициентов из правой части системы
и найдем его значение:

Сделаем аналогичное действие, заменив в первом определителе второй столбец:

Применим формулы Крамера и найдем значения переменных:

и .

Ответ:
Замечание: Этим методом можно решать системы и большей размерности.

Замечание: Если получается, что , а делить на ноль нельзя, то говорят, что система не
имеет единственного решения. В этом случае система имеет или бесконечно много решений или
не имеет решений вообще.

Пример 2 (бесконечное количество решений): Решить систему уравнений:

относительно переменных х и у.
Решение: Найдем определитель матрицы, составленный из коэффициентов системы :

Применять формулы Крамера нельзя. Решим эту систему методом подстановки Решение систем
методом подстановки.

Первое из уравнений системы — равенство, верное при любых значениях переменных (потому
что 4 всегда равно 4). Значит, остается только одно уравнение. Это уравнение связи между
переменными .
Получили, решением системы являются любые пары значений переменных, связанных между
собой равенством .
Общее решение запишется так:
Частные решения можно определять выбирая произвольное значение у и вычисляя х по этому
равенству связи.

и т.д.
Таких решений бесконечно много.
Ответ: общее решение

Частные решения:

Пример 3 (решений нет, система несовместна): Решить систему уравнений:

Решение: Найдем определитель матрицы, составленный из коэффициентов системы :

Применять формулы Крамера нельзя. Решим эту систему методом подстановки

Второе уравнение системы — равенство, неверное ни при каких значениях переменных (конечно
же, так как -15 не равно 2). Если одно из уравнений системы не верно ни при каких значениях
переменных, то и вся системы не имеет решений.
Ответ: решений нет

Метод Гаусса решения систем уравнений.

Метод Гаусса для решения систем линейных алгебраических уравнений заключается в


следующем:
Дано: Решить систему уравнений методом Гаусса:
Запишем расширенную матрицу системы:

И линейными алгебраическими преобразованиями приводим эту матрицу сначала к


верхнетреугольному виду (прямой путь):

Потом к диагональному виду (обратный путь):

И к такому виду, чтобы в правой части этой матрицы стояла единичная:

Тогда решением системы будет:

Замечание: этот метод можно применять к системам любой размерности.


Пример: Решить методом Гаусса систему уравнений:

(Д.К. Фаддеев, И.С. Соминский, Задачи по высшей алгебре, № 400, а )


Решение: Составим расширенную матрицу системы

От элементов третьей строки отнимем элементы второй строки (III +II(-1) )


Умножим элементы первой строки на -3, а второй на 2 ( -3*I; 2*II )

Прибавим к элементам второй строки элементы первой ( II+I )

Поделим третьей строки на -6 ( III:(-6) )

Поменяем для удобства вторую и третью строки местами

Прибавим к третьей строке элементы второй, умноженные на -11 (III+II(-11) )

Матрица коэффициентов приведена к верхнетреугольному виду, т.е. прямой ход закончен.


Сделаем обратный ход, теперь пойдем не «сверху вниз», а «снизу вверх»
Все, обратный ход также закончен, в левой части матрицы стоит единичная, запишем ответ:

Или просто:
Ответ:

Тема . Действия над комплексными числами в алгебраической форме.

Сложение комплексных чисел

Пример 1: Сложить два комплексных числа ,


Для того чтобы сложить два комплексных числа нужно сложить их действительные и мнимые
части:

Просто, не правда ли? Действие настолько очевидно, что не нуждается в дополнительных


комментариях.
Таким нехитрым способом можно найти сумму любого количества слагаемых:
просуммировать действительные части и просуммировать мнимые части.

Для комплексных чисел справедливо правило первого класса: – от


перестановки слагаемых сумма не меняется.
Вычитание комплексных чисел

Пример 2: Найти разности комплексных чисел и , если ,


Действие аналогично сложению, единственная особенность состоит в том, что вычитаемое
нужно взять в скобки, а затем – стандартно раскрыть эти скобки со сменой знака:

Результат не должен смущать, у полученного числа две, а не три части. Просто


действительная часть – составная: . Для наглядности ответ можно переписать
так: .
Рассчитаем вторую разность:

Здесь действительная часть тоже составная:


Чтобы не было какой-то недосказанности, приведу короткий пример с «нехорошей» мнимой
частью: . Вот здесь без скобок уже не обойтись.
Умножение комплексных чисел
Настал момент познакомить вас со знаменитым равенством:

Пример 3: Найти произведение комплексных чисел ,


Очевидно, что произведение следует записать так:

Что напрашивается? Напрашивается раскрыть скобки по правилу умножения многочленов.


Так и нужно сделать! Все алгебраические действия вам знакомы, главное, помнить,
что и быть внимательным.
Повторим, omg, школьное правило умножения многочленов: Чтобы умножить многочлен на
многочлен нужно каждый член одного многочлена умножить на каждый член другого
многочлена.
Я распишу подробно:

Надеюсь, всем было понятно, что


Внимание, и еще раз внимание, чаще всего ошибку допускают в знаках.
Как и сумма, произведение комплексных чисел перестановочно, то есть справедливо
равенство: .
В учебной литературе и на просторах Сети легко найти специальную формулу для
вычисления произведения комплексных чисел. Если хотите, пользуйтесь, но мне кажется, что
подход с умножением многочленов универсальнее и понятнее. Формулу приводить не буду,
считаю, что в данном случае – это забивание головы опилками.
Деление комплексных чисел

Пример 4: Даны комплексные числа , . Найти частное .


Составим частное:

Деление чисел осуществляется методом умножения знаменателя и числителя на


сопряженное знаменателю выражение.

Вспоминаем бородатую формулу и смотрим на наш знаменатель: .


В знаменателе уже есть , поэтому сопряженным выражением в данном случае
является , то есть

Согласно правилу, знаменатель нужно умножить на , и, чтобы ничего не изменилось,


домножить числитель на то же самое число :

Далее в числителе нужно раскрыть скобки (перемножить два числа по правилу,


рассмотренному в предыдущем пункте). А в знаменателе воспользоваться
формулой (помним, что и не путаемся в знаках!!!).
Распишу подробно:

Пример я подобрал «хороший», если взять два числа «от балды», то в результате деления

почти всегда получатся дроби, что-нибудь вроде .


В ряде случаев перед делением дробь целесообразно упростить, например, рассмотрим

частное чисел: . Перед делением избавляемся от лишних минусов: в числителе и в


знаменателе выносим минусы за скобки и сокращаем эти

минусы: . Для любителей порешать приведу правильный


ответ:
Редко, но встречается такое задание:
Пример 5: Дано комплексное число . Записать данное число в алгебраической
форме (т.е. в форме ).
Приём тот же самый – умножаем знаменатель и числитель на сопряженное знаменателю
выражение. Снова смотрим на формулу . В знаменателе уже есть
, поэтому знаменатель и числитель нужно домножить на сопряженное выражение , то
есть на :

Пример 6: Даны два комплексных числа , . Найти их сумму, разность,


произведение и частное.
Это пример для самостоятельного решения. Полное решение и ответ в конце урока.
Тема Производная.
Тема . Правила дифференцирования суммы, произведения и частного.
Правило дифференцирования сложной функции.

Пример 1: Продифференцируем функции: .

Заданная функция является отношением 2-х выражений sinx и 2x+1. Применяем правило
дифференцирования дроби:
Также используем правило дифференцирования суммы и вынесения произвольной постоянной за
знак производной:

И в итоге рассмотрим все правила в 1-ом примере.

Пример 2: Найдем производную функции: , где a –


неотрицательное действительное число.

1-е слагаемое:

2-е слагаемое:
3-е слагаемое:

Объединяем:

Пример 2: Найти производную функции


Это пример для самостоятельного решения (ответ в конце урока).

Пример 3: Найти производную функции


Как всегда записываем:

Разбираемся, где у нас внешняя функция, а где внутренняя. Для этого пробуем (мысленно или
на черновике) вычислить значение выражения при . Что нужно выполнить в
первую очередь? В первую очередь нужно сосчитать чему равно основание: ,
значит, многочлен – и есть внутренняя функция:

И, только потом выполняется возведение в степень , следовательно, степенная функция –


это внешняя функция:

Согласно формуле , сначала нужно найти производную от внешней функции,


в данном случае, от степени. Разыскиваем в таблице нужную формулу: .
Повторяем еще раз: любая табличная формула справедлива не только для «икс», но и для
сложного выражения. Таким образом, результат применения правила дифференцирования
сложной функции следующий:
Снова подчеркиваю, что когда мы берем производную от внешней функции , внутренняя
функция у нас не меняется:

Теперь осталось найти совсем простую производную от внутренней функции и немного


«причесать» результат:

Готово.

Пример 4: Найти производную функции


Это пример для самостоятельного решения (ответ в конце урока).
Для закрепления понимания производной сложной функции приведу пример без
комментариев, попробуйте самостоятельно разобраться, порассуждать, где внешняя и где
внутренняя функция, почему задания решены именно так?

Пример 5 : а) Найти производную функции

б) Найти производную функции

Пример 6: Найти производную функции


Здесь у нас корень, а для того, чтобы продифференцировать корень, его нужно представить в

виде степени . Таким образом, сначала приводим функцию в надлежащий для


дифференцирования вид:

Анализируя функцию, приходим к выводу, что сумма трех слагаемых – это внутренняя
функция, а возведение в степень – внешняя функция. Применяем правило
дифференцирования сложной функции :

Степень снова представляем в виде радикала (корня), а для производной внутренней функции
применяем простое правило дифференцирования суммы:
Готово. Можно еще в скобках привести выражение к общему знаменателю и записать всё
одной дробью. Красиво, конечно, но когда получаются громоздкие длинные производные –
лучше этого не делать (легко запутаться, допустить ненужную ошибку, да и преподавателю
будет неудобно проверять).

Пример 7: Найти производную функции


Это пример для самостоятельного решения (ответ в конце урока).
Интересно отметить, что иногда вместо правила дифференцирования сложной функции

можно использовать правило дифференцирования частного , но такое решение


будет выглядеть как извращение необычно. Вот характерный пример:

Пример 8: Найти производную функции

Здесь можно использовать правило дифференцирования частного , но гораздо


выгоднее найти производную через правило дифференцирования сложной функции:

Подготавливаем функцию для дифференцирования – выносим минус за знак производной, а


косинус поднимаем в числитель:

Косинус – внутренняя функция, возведение в степень – внешняя функция.


Используем наше правило :

Находим производную внутренней функции, косинус сбрасываем обратно вниз:


Готово. В рассмотренном примере важно не запутаться в знаках. Кстати, попробуйте решить

его с помощью правила , ответы должны совпасть.

Пример 9: Найти производную функции


Это пример для самостоятельного решения (ответ в конце урока).
До сих пор мы рассматривали случаи, когда у нас в сложной функции было только одно
вложение. В практических же заданиях часто можно встретить производные, где, как
матрешки, одна в другую, вложены сразу 3, а то и 4-5 функций.

Пример 10: Найти производную функции

Разбираемся во вложениях этой функции. Пробуем вычислить выражение с помощью


подопытного значения . Как бы мы считали на калькуляторе?

Сначала нужно найти , значит, арксинус – самое глубокое вложение:

Затем этот арксинус единицы следует возвести в квадрат :

И, наконец, семерку возводим в степень :

То есть, в данном примере у нас три разные функции и два вложения, при этом, самой
внутренней функцией является арксинус, а самой внешней функцией – показательная
функция.
Начинаем решать
Согласно правилу сначала нужно взять производную от внешней функции.
Смотрим в таблицу производных и находим производную показательной
функции: Единственное отличие – вместо «икс» у нас сложное
выражение , что не отменяет справедливость данной формулы. Итак, результат
применения правила дифференцирования сложной функции следующий:

Под штрихом у нас снова сложная функция! Но она уже проще. Легко убедиться, что
внутренняя функция – арксинус, внешняя функция – степень. Согласно правилу
дифференцирования сложной функции сначала нужно взять производную от степени:

Теперь все просто, находим по таблице производную арксинуса и немного «причесываем»


выражение:
Тема .Неопределенный интеграл. Определенный интеграл. Вычисление
определенных интегралов. Вычисление площадей плоских фигур.

Пример

Задание. Вычислить неопределенный интеграл


Решение. Для решения данного интеграла не нужно использовать свойства неопределенных
интегралов, достаточно формулы интеграла степенной функции:

В нашем случае , тогда искомый интеграл равен:

Ответ.
Пример

Задание. Вычислить неопределенный интеграл


Решение. Преобразуем подынтегральное выражение. Для этого вынесем из знаменателя за
знак интеграла

далее, используя таблицу интегралов (Формула №11), получим


Ответ.
Пример

Задание. Найти неопределенный интеграл


Решение. Введем замену и полученный интеграл находим как интеграл от
степенной функции:

Сделаем обратную замену

Ответ.
Пример
Интегрирование по частям

Задание. Найти неопределенный интеграл


Решение. Воспользуемся методом интегрирования по частям. Для этого положим

Подставим это в формулу для интегрирования по частям, затем воспользуемся формулой


интеграла косинуса из таблицы интегралов

Ответ.

Пример 1: Вычислить определенный интеграл


Решение:

(1) Выносим константу за знак интеграла.


(2) Интегрируем по таблице с помощью самой популярной формулы .

Появившуюся константу целесообразно отделить от и вынести за скобку. Делать это не


обязательно, но желательно – зачем лишние вычисления?

(3) Используем формулу Ньютона-Лейбница . Сначала


подставляем в верхний предел, затем – нижний предел. Проводим дальнейшие вычисления
и получаем окончательный ответ.

Пример 2: Вычислить определенный интеграл


Это пример для самостоятельно решения, решение и ответ в конце урока.
Немного усложняем задачу:

Пример 3: Вычислить определенный интеграл


Решение:

Пример 1. Вычислить интеграл

Решение: На основании таблицы основных интегралов и формулы (1) имеем:

Пример 2. Вычислить интеграл

Решение: На основании таблицы основных интегралов и формулы (1) имеем:

Пример 3. Вычислить интеграл


Решение: На основании таблицы основных интегралов и формулы (1) имеем:

Пример 4 Вычислить интеграл

Решение: На основании формулы произведения синусов, таблицы основных интегралов и


формулы (1) имеем:

Пример 5. Вычислить интеграл

Решение: Разложим подынтегральную функцию на сумму простых дробей,


Решив систему

Получим

Тогда на основании таблицы основных интегралов и формулы (1) имеем

Пример 6. Вычислить интеграл

Решение: На основании таблицы основных интегралов и формулы (2) имеем:


Сделаем замену ex + 4 = t2, тогда ex= t2– 4, ex dx = 2t dt,
Если x= ln5, то t = 3; если x= ln12, то t = 4. Тогда
МИНИСТЕРСТВО ОБРАЗОВАНИЯ И НАУКИ КЫРГЫЗСКОЙ РЕСПУБЛИКИ
КЫРГЫЗСКИЙ НАЦИОНАЛЬНЫЙ УНИВЕРСИТЕТ
имени Ж. БАЛАСАГЫНА
ПРОФЕССИОНАЛЬНЫЙ КОЛЛЕДЖ ОТДЕЛЕНИЕ ПЕРЕВОДЧЕСКОЕ ДЕЛО

МАТЕРИАЛЫ ДЛЯ ОРГАНЗАЦИИ САМОСТОЯТЕЛЬНОЙ РАБОТЫ СТУДЕНТОВ

ДИСЦИПЛИНЫ

Профессиональная математика
Специальность « Переводческое дело » , «Туризм»

Тематика, формы выполнения и критерии оценки СРС


Критерии оценки СРС
Кол-во баллов
Параметры оценивания (Максимальный
балл-20)
1-модуль
1.Понимание содержания СРС (реферата, эссе и др.), через
2
четкую формулировку целей и задач.
2. Умения студента использовать теоретические знания при
2
выполнении практических задач
3.Умение ориентироваться в потоке информации, выделять
6
главное;
Итого за 1-модуль 10
2-модуль
4. Умение четко сформулировать проблему, предложив ее
3
решение, критически оценить решение и его последствия
5.Умение показать, проанализировать альтернативные 3
возможности, варианты действий;
6. Умение сформировать свою позицию, оценку и 2
аргументировать ее.
7. Сформированность обще учебных умений; 2
Итого за 2-модуль 10

Задачи для самостоятельной работы студентов


Решение задач по темам: операции над матрицами, вычисление определителей,
миноры и алгебраические дополнения, вычисление обратной матрицы, ранг матрицы,
применения в экономике.

1. Найти линейные комбинации матриц

A=¿ ( 1 2 3 ¿ ) ¿ ¿¿ B=¿ (−2 3 0 ¿ ) ¿ ¿¿


а) 2А+3В, где ¿ , ¿ .

A=¿( 2−1 0¿) ( 3 4−2¿) ¿¿¿ B=¿( 3 1 2¿)(−2 1 3¿)¿¿¿


б) 4А – 5В, где ¿ , ¿ .
A=¿( 7−2 3−4¿) ( 0 2 1−1¿) ¿¿¿ B=¿(2 −1−3 1¿)(7 −1 0 4¿) ¿¿¿
в) 3А+4В, где ¿ , ¿ .
2. Найти произведения АВ и ВА (если это возможно).

A=¿ ( 3 −2 ¿ ) ¿ ¿ ¿
а) ¿

A=¿ ( 1 2 3 ¿ ) ¿ ¿¿ B=¿(3 4 5¿)(6 0−2¿)¿¿¿


б) ¿ , ¿ .
A=¿ ( 1 2 ¿ ) ¿ ¿ ¿
в) ¿ .
3. Предприятие производит продукцию трех видов и использует сырье двух типов. Нормы затрат

сырья на единицу продукции каждого вида заданы матрицей


A= 2 1 3 .
1 3 4 Стоимость ( )
единицы сырья каждого типа задана матрицей В=(10 15). Каковы общие затраты
предприятия на производство 100 единиц продукции первого вида, 200 единиц продукции
второго вида и 150 единиц продукции третьего вида?

4. Вычислите определители второго порядка.

|3 −2¿|¿¿¿ |2 3¿|¿ ¿¿ |3 −2¿|¿ ¿¿ |√ а −1 ¿|¿ ¿¿ | sinα cosα ¿|¿ ¿¿


¿ , б) ¿ , в) ¿ , г) ¿ , д) ¿
а)

5. Вычислите определители третьего порядка.

|3−2 1¿| −2 2 3¿|¿¿¿


|1 2 0¿| 0 1 3¿|¿¿ |2−1 3¿| −2 3 2¿|¿¿¿ |а 1 а¿|−1 а 1¿|¿¿¿
а) ¿ б) ¿ , , в) ¿ , г) ¿

A=(
6 3 ) . Найти все миноры и алгебраические дополнения.
5 7
6. Дана матрица

( )
5 −2 3
А= 2 −1 0
7. Дана матрица 1 1 2 . Найти все миноры и алгебраические дополнения.

8. Вычислите определители, используя разложение по строке, либо по столбцу.


1 2 3 2
2 5 7 1 0 −1 0 1 −2 3
| |
|0 1 2 | |7 −7 21 | −1 −1 0 4
а) 0 2 4 ; б) 4 8 12 ; в) 2 3 −2 5 .
−1
9. Найти обратную матрицу A

( ) ( )
1 2 1 5 −2 3

а)
A= (56 73 ) ; б)
А= 2 4 2
1 1 2 ; в)
А= 2 −1 0
1 1 2 .
Решение задач по темам: системы линейных уравнений и нахождение их решений
методами Крамера , Гаусса, обратной матрицы.

1.Решите системы методом Крамера и матричным методом :

а)
{ 2 х −4 y + 3 z = 1 ¿ { x −2 y + 4 z =3 ¿ ¿ { х + 2 y + 3 z =5 ¿ { 2 x − y − z =1 ¿ ¿ { х + y + z =a ¿ { x − y + z =b ¿ ¿
б) в)

2. Методом Гаусса решите следующие линейные системы:

а)
{х1+х2+х3=8¿{5x1+2х2+4х3=14¿¿ ¿ б)
{х1+2х2−х3=2¿{2x1−6х2+х3=−11¿¿ ¿
в)
{ x1+3x2−x3=2 ¿ {2 x1−x2+x3+3x4=14 ¿ ¿¿¿¿
г)
{2 x 1 +7 x 2 +3 x 3 + x 4 = 5 ¿ { x 1 + 3 x 2 + 5 x 3 − 2 x 4 = 3 ¿ { x 1 + 5 x 3 − 9 x 3 + 8 x 4 = 1 ¿ ¿
д)
{ x1−2x2+x4=−3¿{3x1−x2−2x3=1¿{2x1+x2−2x3−x4=4 ¿ ¿
е)
{x1+2x2+3x3=6¿{2x1−3x2+x3=0¿{3x1−2x2+4x3=5 ¿ ¿ .

7. Найти производные следующих функций:


2 3 1+ln x
y= −3
a) һy=5 x −3 √ x +7 / x +4 ;
7 y=
4 3 5
b) √ x √ x c) y=( x 4 +1 ) / ( x 4 −1 ) ; d) x e)


2
cos x +1
y=
y=( sin2 x ) / ( x 3 +1 ) f) sin 2 x+ 1 g) y=( x +3 x−1 ) ;
5 4
h) y= √ x 4 +sin 4 x
3
k)
3 1
3 arctg √1+ x2 tg 5x y=
y=( 2 x −tg x )
4 4

m) y=e n) y=3 o) y= (4+3 x ) √


3 2
q) (1−x 2 )5 r)
1+ √ x +1 2
y=ln
y=ln( x 2 +2 x ) 2
s) y=cosln(1−x ) t) x u) y= x +x √ x √
5 3

v) y=e tg 4 x ;
x
w) y=x cos x z) y=e
2 x
√ 1−e 2x x
+arcsin e
8. Используя логарифмическую производную найти производные следующих функций:
1
x2 tg 2 x
e) y=( x +1 )
cos 5 x 3
b) y=x d) y= ( sin3 x )
x x
a) y=(sin x ) c) y=(cos x ) .

''
9. Найти производную второго порядка y функций
a) y=( 1+ 4 x ) arctg 2 x b) y=( x +1)ln(1+x )
2 2 2
3 2
10.Найти значения производных любого порядка функции y=x −5 x +7 x−2 в точке
x=2 .
2x
11.Удовлетворяет ли функция y=C 1 e +C 2 e 3 x при любых постоянных C 1 и C 2
'' '
уравнению y −5 y +6 y =0 .

Решение задач по теме: вычисление интеграла.


Найти указанные неопределенные интегралы и результаты проверить
дифференцированием
1)Методом непосредственного интегрирования

a)
∫ ex
( 1−
e− x
x 2 )b) √
1
dx ∫ 3 2 −
x
1
x√x (
dx
c)
x−2
√ x 3
dx ) ∫
2) Методом замены переменной
e√
x x
∫ dx
∫ tgx dx ∫ e
cos x
sin x dx
∫ x dx
d) √
2
a) x +1 b) c)
dx x+1
∫ √ 1+ln x
x
dx ∫
√ x 2+2x+3 g) ∫ x2+3dxx+3 ∫ √3 3 x+1 dx
e) f) h)
sin √ x x 2 dx
1−2 x x dx ∫ √ x
6 dx ∫ 6 3
3 2
k)∫ sin x cos x dx m)∫ √
2 3
n) o) x +2 x +3
3) Методом интегрирования по частям
x


a) x ln x−1 dx b)
( ) √ x ln x dx c) x e ∫ ∫ 2 2
dx
4) Вычислить определенные интегралы.
e3 √3
dx
∫ ∫ x 5 √ 1+ x2 dx .
2

a)

1
( 2 x2 +
2
x
4)dx .
b) 1
x √ 1+ ln x
.
c) 1
1 8
∫ xe−x dx ∫ √xdx
1+x
d) 0 e) 3
5) Вычислить несобственные интегралы.
+∞ ∞ ∞
dx dx
∫ 2
−∞ x +1
∫ x ln x
, ( a>1 ) ∫ sin xdx
a) b) a c) 0
Контрольные вопросы итогового контроля.
1. Что такое множество?
2. Пустое, конечное и бесконечное множество.
3. Объединение множеств.
4. Пересечение множеств.
5. Разность множеств.
6. Диаграмма Эйлера- Венна.
7. Свойства объединения и пересечения множеств
8. Число элементов объединения и разности двух конечных множеств
9. Прямоугольная система координат.
10. Расстояние между двумя точками.
11. Деления отрезка в данном отношении.
12. Площадь треугольника
13. Уравнения прямых.
14. Общее уравнение прямых.
15. Уравнение прямых с угловым коэффициентом.
16. Расстояние от точки до прямой.
17. Условие параллельности и перпендикулярности прямых.
18. Постоянные и переменные
19. Представление функции формулой и таблицей
20. Обозначение функций. Координаты. Графическое представление функций
21. Монотонная функция. Ограниченная и неограниченная функции.
22. Непрерывная и разрывная функции.
23. Чётная и нечётная функции.
24. Предел последовательности и функции.
25. Свойство пределов.
26. Первый и второй замечательный предел.
27. Производная.
28. Геометрический смысл производной.
29. Уравнение касательной.
30. Механический смысл производной.
31. Основные свойства производных и дифференциалов
32. Производные элементарных функций
33. Основные определения и свойства интегралов
34. Свойства неопределенного интеграла:
35. Таблица интегралов
36. Методы интегрирования неопределенных интегралов
37. Определенный интеграл
38. Формула Ньютона-Лейбница
39. Объем тела вращения
40. Методы интегрирования определенного интеграла.
МИНИСТЕРСТВО ОБРАЗОВАНИЯ И НАУКИ КЫРГЫЗСКОЙ РЕСПУБЛИКИ
КЫРГЫЗСКИЙ НАЦИОНАЛЬНЫЙ УНИВЕРСИТЕТ
имени Ж. БАЛАСАГЫНА
ПРОФЕССИОНАЛЬНЫЙ КОЛЛЕДЖ ОТДЕЛЕНИЕ ПЕРЕВОДЧЕСКОЕ ДЕЛО

КОНТРОЛЬНО-ИЗМЕРИТЕЛЬНЫЕ МАТЕРИАЛЫ

ДИСЦИПЛИНЫ

Профессиональная математика

Контрольные тестовые задания для проведения рубежного контроля

Тестовые задания

Кыргызский национальный университет им. Ж. Баласагына


Профессиональный колледж
Дисциплина: «Профессиональная математика»
ЭКЗАМЕНАЦИОННЫЙ БИЛЕТ №1 Критерий оценки
1.Матрица. Определение обратной матрицы. 10 баллов
2.Определение комплексного числа . 10 баллов

( )( )
301 2

3.Найдите произведение матриц : −1 0 5 3 20 баллов
11 2 1
Итого за задание 40 баллов

Кыргызский национальный университет им. Ж. Баласагына


Профессиональный колледж

Дисциплина: «Профессиональная математика»


ЭКЗАМЕНАЦИОННЫЙ БИЛЕТ №2 Критерий оценки
1. Геометрические смысл комплексного числа. 10 баллов
2. Матрица. Виды матрицы. 10 баллов

( )( )
301 2 20 баллов

3.Найдите произведение матриц : −1 0 5 3
11 2 1

Итого за задание 40 баллов

Кыргызский национальный университет им. Ж. Баласагына


Профессиональный колледж
Дисциплина: «Профессиональная математика»
ЭКЗАМЕНАЦИОННЫЙ БИЛЕТ №3 Критерий оценки
1.Свойства определители.. 10 баллов
2.Производная. Определения производной. 10 баллов

( )( )
301 2 20 баллов

3.Найдите произведение матриц : −1 0 5 3
11 2 1

Итого за задание 40 баллов


Кыргызский национальный университет им. Ж. Баласагына
Профессиональный колледж

Дисциплина: «Профессиональная математика»


ЭКЗАМЕНАЦИОННЫЙ БИЛЕТ №4 Критерий оценки
1.СЛАУ.Метод Гаусса. 10 баллов
10 баллов
2.Матрица. Действия над матрицами.
20 баллов
3.Методом замены переменной

∫ x2x+1 dx
Итого за задание 40 баллов
Кыргызский национальный университет им. Ж. Баласагына
Профессиональный колледж

Дисциплина: «Профессиональная математика»


ЭКЗАМЕНАЦИОННЫЙ БИЛЕТ №5 Критерий оценки
1.СЛАУ. Метод Крамера. 10 баллов
2.Матрица.действие над матрицами. 10 баллов
''
3.Найти производную второго порядка y функций 20 баллов
y=( 1+ 4 x 2 ) arctg 2 x

Итого за задание 40 баллов

Кыргызский национальный университет им. Ж. Баласагына


Профессиональный колледж

Дисциплина: «Профессиональная математика»


ЭКЗАМЕНАЦИОННЫЙ БИЛЕТ №6 Критерий оценки
1.Миноры и алгебраические дополнения. 10 баллов
2.Определенные интегралы. 10 баллов
''
y
3.Найти производную второго порядка функций
2 2 20 баллов
y=( x +1)ln(1+x )
Итого за задание 40 баллов

Кыргызский национальный университет им. Ж. Баласагына


Профессиональный колледж

Дисциплина: «Профессиональная математика»


ЭКЗАМЕНАЦИОННЫЙ БИЛЕТ №7 Критерий оценки
1.Элементы комбинаторики. 10 баллов
2.Что называется матрицей, матрицей- строкой, 10 баллов
матрицей- столбцом, вектором?
3. Вычислить определенные интегралы. 20 баллов
√3
∫ x √5 1+ x ² d x
1

Итого за задание 40 баллов

Кыргызский национальный университет им. Ж. Баласагына


Профессиональный колледж

Дисциплина: «Профессиональная математика»


ЭКЗАМЕНАЦИОННЫЙ БИЛЕТ №8 Критерий оценки
1.Комплексные числа. Тригонометрическая форма 10 баллов
комплексного числа.
2. Элементы комбинаторики. 10 баллов
3. Вычислить определенные интегралы. 20 баллов
2

( )
∫ 2 x2 + 24 dx .
1 x
Итого за задание 40 баллов

Кыргызский национальный университет им. Ж. Баласагына


Профессиональный колледж

Дисциплина: «Профессиональная математика»


ЭКЗАМЕНАЦИОННЫЙ БИЛЕТ №9 Критерий оценки
1. Какая матрица называется обратной по отношению к 10 баллов
данной?
2. Действия над комплексными числами. 10 баллов
3. Произвести умножение комплексных чисел: 20 баллов
(−2+3 i ) (3+5 i)
Итого за задание 40 баллов

Кыргызский национальный университет им. Ж. Баласагына


Профессиональный колледж

Дисциплина: «Профессиональная математика»


ЭКЗАМЕНАЦИОННЫЙ БИЛЕТ №10 Критерий оценки
1. Действия над матрицами. 10 баллов
2. Определение неопределенного интеграла. 10 баллов
3.Найти производные функций. 20 баллов
4
y=3 x ³+5 √ х − 3
3 5

х
Итого за задание 40 баллов

Кыргызский национальный университет им. Ж. Баласагына


Профессиональный колледж

Дисциплина: «Профессиональная математика»


ЭКЗАМЕНАЦИОННЫЙ БИЛЕТ №11 Критерий оценки
1. Сложение матрицы. 10 баллов
2. Интеграл.Неопределенный интеграл. 10 баллов
3. Найти производную функцию: 20 баллов
2
2 x −6
y=
x−2
Итого за задание 40 баллов

Кыргызский национальный университет им. Ж. Баласагына


Профессиональный колледж

Дисциплина: «Профессиональная математика»


ЭКЗАМЕНАЦИОННЫЙ БИЛЕТ №12 Критерий оценки
1. Правила дифференцирования алгебраической суммы, 10 баллов
произведения и частного.
2. Комплексные числа. 10 баллов
3. Найти производную функцию: 20 баллов
x3
y=
x 2 −4 ;
Итого за задание 40 баллов
Кыргызский национальный университет им. Ж. Баласагына

Профессиональный колледж

Дисциплина: «Профессиональная математика»


ЭКЗАМЕНАЦИОННЫЙ БИЛЕТ №13 Критерий оценки
1. Правила треугольника. 10 баллов
2. Свойства определителей. 10баллов
20 баллов
3. Найти линейные комбинации матриц

A=¿( 2−1 0¿) ( 3 4−2¿) ¿¿¿ B=¿( 3 1 2¿)(−2 1 3¿)¿¿¿


4А – 5В, где ¿ , ¿ .
Итого за задание 40 баллов

Кыргызский национальный университет им. Ж. Баласагына


Профессиональный колледж

Дисциплина: «Профессиональная математика»


ЭКЗАМЕНАЦИОННЫЙ БИЛЕТ №14 Критерий оценки
1. Теория вероятности. 10 баллов
2. Неопределенный интеграл.. 10 баллов

( )( )
301 2 20 баллов

3.Найдите произведение матриц : −1 0 5 3
11 2 1

Итого за задание 40 баллов

Кыргызский национальный университет им. Ж. Баласагына


Профессиональный колледж

Дисциплина: «Профессиональная математика»


ЭКЗАМЕНАЦИОННЫЙ БИЛЕТ №15 Критерий оценки
1. Определение производной. 10 баллов
2. Общее правило нахождения производной 10 баллов

( )( )
301 2 20 баллов

3.Найдите произведение матриц : −1 0 5 3
11 2 1

Итого за задание 40 баллов


Кыргызский национальный университет им. Ж. Баласагына
Профессиональный колледж
Дисциплина: «Профессиональная математика»
ЭКЗАМЕНАЦИОННЫЙ БИЛЕТ №16 Критерий оценки
1. Неопределенный интеграл и его свойства. 10 баллов
2. Определение матрицы. Виды матриц. 10 баллов
3. Решите системы методом Крамера и матричным методом : 20 баллов

{ 2 х −4 y + 3 z = 1 ¿ { x −2 y + 4 z =3 ¿ ¿
Итого за задание 40 баллов

Кыргызский национальный университет им. Ж. Баласагына


Профессиональный колледж
Дисциплина: «Профессиональная математика»
ЭКЗАМЕНАЦИОННЫЙ БИЛЕТ №17 Критерий оценки
1. Методы интегрирования определенного интеграла. 10 баллов
2. Элементы комбинаторики. 10 баллов
3. Найти линейные комбинации матриц 20 баллов

A=¿ ( 1 2 3 ¿ ) ¿ ¿¿ B=¿ (−2 3 0 ¿ ) ¿ ¿¿


2А+3В, где ¿ , ¿ .
Итого за задание 40 баллов

Кыргызский национальный университет им. Ж. Баласагына


Профессиональный колледж
Дисциплина: «Профессиональная математика»
ЭКЗАМЕНАЦИОННЫЙ БИЛЕТ №18 Критерий оценки
1. Производная. 10 баллов
2. Основные определения и свойства интегралов. 10 баллов
3. Найти линейные комбинации матриц 20 баллов

A=¿( 2−1 0¿) ( 3 4−2¿) ¿¿¿ B=¿( 3 1 2¿)(−2 1 3¿)¿¿¿


4А – 5В, где ¿ , ¿ .
Итого за задание 40 баллов

Кыргызский национальный университет им. Ж. Баласагына


Профессиональный колледж
Дисциплина: «Профессиональная математика»
ЭКЗАМЕНАЦИОННЫЙ БИЛЕТ №19 Критерий оценки
1. Таблица интегралов 10 баллов
2. Дифференциал функции. 10 баллов
3. Найти линейные комбинации матриц 20 баллов

A=¿( 2−1 0¿) ( 3 4−2¿) ¿¿¿ B=¿( 3 1 2¿)(−2 1 3¿)¿¿¿


4А – 5В, где ¿ , ¿ .
Итого за задание 40 баллов

Кыргызский национальный университет им. Ж. Баласагына


Профессиональный колледж
Дисциплина: «Профессиональная математика»
ЭКЗАМЕНАЦИОННЫЙ БИЛЕТ №20 Критерий оценки
1. Комплексное числа. 10 баллов
2. Основные определения и свойства интегралов. 10 баллов
20 баллов

3. Найти линейные комбинации матриц

A=¿( 2−1 0¿) ( 3 4−2¿) ¿¿¿ B=¿( 3 1 2¿)(−2 1 3¿)¿¿¿


4А – 5В, где ¿ , ¿ .
Итого за задание 40 баллов
МИНИСТЕРСТВО ОБРАЗОВАНИЯ И НАУКИ КЫРГЫЗСКОЙ РЕСПУБЛИКИ
КЫРГЫЗСКИЙ НАЦИОНАЛЬНЫЙ УНИВЕРСИТЕТ
имени Ж. БАЛАСАГЫНА
ПРОФЕССИОНАЛЬНЫЙ КОЛЛЕДЖ ОТДЕЛЕНИЕ ПЕРЕВОДЧЕСКОЕ ДЕЛО

СПИСОК ЛИТЕРАТУРЫ

ДИСЦИПЛИНЫ

Профессиональная математика
Специальность « Переводческое дело » , «Туризм»

Список литературы
ОСНОВНАЯ ЛИТЕРАТУРА
1.Туганбаев У.М.,Туганбаев М.М. «Высшая математика» Учебник для ВУЗои -2005.
2.Дмитрий Письменный.»Корспект лекций по высшей МАТЕМАТИКЕ» 1часть,
Москва АЙРИС ПРЕСС 2017г.
3. Усубакунов Т.»Дифференциалдык тендемелер».-Ф:Жогорку математика.,1988
4. И.Л. Соловейчик, В.Т. Лисичкин. Сборник задач по математике для техникумов.
5. Минорский В.П., Сборник задач по высшей математике.
6.Демидович Б. П. Сборник задач и упражнений по математическому анализу .-М.:
Наука, 1990.-624с.
7 Высшая математика . Учебное пособие. В.Л.Ким,В.Бостериева,Е.С.Федорова. Б.1998.

.
ДОПОЛНИТЕЛЬНАЯ ЛИТЕРАТУРА
1.Тарасов Н.П. Курс высшей математики. -М: Наука, 1975.-448 с.
2.Гроссман С., Тернер Дж. Математика для биологов. -М.: Высшая школа, 1983.
3.Фихтенгольц Г. М. Основы математического анализа. Т.1.-М.: Наука, 1968.-440с.
4.Пискунов Н.С. Дифференциальное и интегральное исчисления. М.: Наука, 1978.-575с.
5.Шифр № В11/Б-42 Бекбоев И.Б. Жогорку математиканын жалпы курсу. Бишкек. 2000
6.Шифр № В11/0579 Омурбаев Н.Т. Жогорку математиканын жалпы элементтери. Бишкек
2012
7.Шифр № кырг.В16/Э-40 Урдалетова А.Б, Йылмаз Ж. Экономикалык жана коомдук
илимдер учун математика . Бишкек 2010.
8. Демидович Б. П. Сборник задач и упражнений по математическому анализу.-М.: Наука,
1990.-624с.
9.Лобоцкая Н.Л., Морозов Ю.В., Дунаев А.А. Высшая математика. Минск: Высшая школа,
1987.-319 с.
10.Минорский В.П., Сборник задач по высшей математике

Электронные ресурсы:
8 file:///E:/проф.матем/матем.книга%20для%20СПО_Глотова_Самохвалова-2016г.и..pdf
9 www.okuma.kg
10 https ; //clck.ru/34Srat
11 https://fileskachat.com/download/6647_587665e51b67eb5404f4da219e6cc079.html
12 http://www.bymath.net/ - элементарная математика
13 http://graphfunk.narod.ru — графики элементарных функций
14 http://window.edu.ru/window — информационная система «Единое окно доступа к
образовательным ресурсам» с обширной библиотекой по основным
разделам
15 математики.
7.http://www.exponenta.ru/ - образовательный математический сайт
МИНИСТЕРСТВО ОБРАЗОВАНИЯ И НАУКИ КЫРГЫЗСКОЙ РЕСПУБЛИКИ
КЫРГЫЗСКИЙ НАЦИОНАЛЬНЫЙ УНИВЕРСИТЕТ
имени Ж. БАЛАСАГЫНА

ПРОФЕССИОНАЛЬНЫЙ КОЛЛЕДЖ ОТДЕЛЕНИЕ ПЕРЕВОДЧЕСКОЕ ДЕЛО

ГЛОССАРИЙ

ДИСЦИПЛИНЫ «Профессиональная математика»

Специальность « Переводческое дело » , «Туризм»


ГЛОССАРИЙ

Абсцисса (лат. слово abscissa - «отрезанная»). Заимств. из франц. яз. в начале 19 в. Франц. abscisse
– из лат. Это одна из декартовых координат точки, обычно первая, обозначаемая буквой x. В
современном смысле Т. употреблен впервые немецким ученым Г. Лейбницем (1675).

Аксиома (греч. слово axios- ценный; axioma – «принятие положения», «почет», «уважение»,
«авторитет»). В рус.яз. – с Петровских времен. Это основное положение, самоочевидный принцип.
Впервые Т. встречается у Аристотеля. Использовался в книгах Евклида «Начала». Большую роль
сыграли работы древнегреческого ученого Архимеда, который сформулировал аксиомы,
относящиеся к измерению величин. Вклад в аксиоматику внесли Лобачевский, Паш, Пеано.
Логически безупречный список аксиом геометрии был указан немецким математиком Гильбертом
на рубеже 19 и 20 вв.

Апофема (греч. слово apothema,apo – «от», «из»; thema – «приложенное», «поставленное»).


1.В правильном многоугольнике апофема – отрезок перпендикуляра, опущенного из его центра на
любую из его сторон, а также его длина.
2.В правильной пирамиде апофема – высота любой его боковой грани.
3.В правильной усеченной пирамиде апофема – высота любой ее боковой грани.

Аппликата (лат. слово applicata – «приложенная»). Это одна из декартовых координат точки в
пространстве, обычно третья, обозначаемая буквой Z.

Биссектриса (лат. слова bis – «дважды» и sectrix –«секущая»). Заимств. В 19 в. из франц. яз. где
bissectrice – восходит к лат. словосочетанию. Это прямая, проходящая через вершину угла и
делящая его пополам.

Вектор (лат. слово vector – «несущий», «носитель»). Это направленный отрезок прямой, у которой
один конец называют началом вектора, другой конец – концом вектора. Этот термин ввел
ирландский ученый У. Гамильтон (1845).

Вертикальные углы (лат. слова verticalis – «вершинный»). Это пары углов с общей вершиной,
образуемые при пересечении двух прямых так, что стороны одного угла являются продолжением
сторон другого.

Вероятность - числовая характеристика степени возможности появления определенного события


в тех или иных определенных, могущих повторяться неограниченное число раз условиях.

Гексаэдр (греч. слова geks – «шесть» и edra – «грань»). Это шестигранник. Этот Т. приписывают
древнегреческому ученому Паппу Александрийскому (3 век).
Геометрия (греч. слова geо – «Земля» и metreo – «измеряю»). Др.-рус. заимств. из греч.яз. Часть
математики, изучающая пространственные отношения и формы. Т. появился в 5 веке до н.э. в
Египте, Вавилоне.

Геометрический смысл определенного интеграла - определенный интеграл от функции f(х) по


отрезку [a; b] равен площади криволинейной трапеции

Геометрический смысл производной - если функция у = f(х) имеет производную в точке х, тогда
существует касательная к графику этой функции в точке М0(х0;у0), уравнение которой y – у0 =f(х
)(х - х ), где f(х ) = tg , где - угол наклона этой касательной к оси ох.

Гипербола (греч. слово hyperballo – «прохожу через что-либо»). Заимств. в 18 в. из лат. яз. Это
незамкнутая кривая из двух неограниченно простирающихся ветвей. Т.ввел древнегреческий
ученый Апполоний Пермский.
Гипотенуза (греч.слово gyipotenusa – «стягивающая»). Замств. из лат. яз. в 18 в., в котором
hypotenusa – от греч. сторона прямоугольного треугольника, лежащая против прямого угла.
Древнегреческий ученый Евклид (3 век до н.э.) вместо этого термина писал, «сторона, которая
стягивает прямой угол».

Градус (лат. слово gradus – «шаг», «ступень»). Единица измерения плоского угла, равная 1/90
части прямого угла. Измерение углов в градусах появилось более 3 лет назад в Вавилоне.
Обозначения, напоминающие современные, использовались древнегреческими ученым
Птолемеем.

График (греч. слово graphikos- «начертанный»). Это график функции – кривая на плоскости,
изображаемая зависимость функции от аргумента.

Диагональ (греч. слово dia – «через» и gonium – «угол»). Это отрезок прямой, соединяющий две
вершины многоугольника, не лежащие на одной стороне. Т. встречается у древнегреческого
ученого Евклида (3 век до н.э.).
Диаметр (греч. слово diametros – «поперечник», «насквозь», «измеряющий» и слово dia – «между»,
«сквозь»). Т. «деление» в русском языке впервые встречаются у Л.Ф.Магницкий.

Дифференциал (лат. слово differento- «разность»). это главная часть приращения функции, равная
произведению производной функции у = f(х) на приращение аргумента ∆х: dy=fI(x)*∆x. Так
как ∆х=dx, тоdy=fI(x)*∆x – произведение производной функции у = f(х) на дифференциал
аргумента dx.. Это одно из основных понятий математического анализа. Этот Т. встречается у
немецкого ученого Г. Лейбница в 1675 г. (опубликовано в 1684г.).

Декартова прямоугольная система координат в пространстве - это три взаимно


перпендикулярные прямые: Ося абсцисс (ох), ось ординат (оу) и ось аппликат (oz) и начало
координат (о). Плоскости, проходящие через оси координат, называются координатными. Они
делят пространство на 8 областей – октантов.

Длина вектора - это расстояние между началом и концом вектора. Обозначение:

Достоверное событие - это событие, которое в результате испытания обязательно происходит.


Обозначение: Ω.

Знаменатель - число, показывающее размеры долей единицы, из которых составлена дробь.


Впервые встречается у византийского ученого Максима Плануда (конец 13 века).

Интеграл (лат. слово integro – «восстанавливать» или integer – «целый»). Заимств. во второй
половине 18 в. из франц. яз. на базе лат. integralis – «целый», «полный». Одно из основных
понятий математического анализа, возникшее в связи потребностью измерять площади, объемы,
отыскивать функции по их производным. Обычно эти концепции интеграла связывают с
Ньютоном и Лейбницем. Впервые это слово употребил в печати швец. Ученый Я. Бернулли (1690
г.). Знак ∫ - стилизованная буква S от лат. слова summa – «сумма». Впервые появился у Г. В.
Лейбница.

Интервал (лат. слово intervallum – «промежуток», «расстояние»). Множество действительных


чисел, удовлетворяющее неравенству a < x

Иррациональное число (т. слово irrationalis – «неразумный»). Число, не являющееся


рациональным. Т. ввел немецк. ученый М.Штифель (1544). Строгая теория иррациональных чисел
была построена во 2-ой половине 19 века.

Испытание (эксперимент) - осуществление определенного комплекса условий.

Исход - результат испытания (событие).


Комбинаторика - лат.слово combinare – «соединять». Раздел математики, в котором изучаются
различные соединения и размещения, связанные с подсчетом комбинаций из элементов данного
конечного множества.

Классическая вероятность события А - это отношение числа N(A) элементарных исходов,


благоприятствующих событию А, к общему числу N всех равновозможных элементарных исходов
испытания.

Коллинеарные векторы - это векторы, лежащие на одной прямой или на параллельных прямых.
Обозначение: .

Компланарные векторы - это векторы, лежащие в одной плоскости или в параллельных


плоскостях.

Комплексное число z - это упорядоченная пара действительных чисел (x;y), первое из


которых x называется действительной частью, а второе число y – мнимой частью.
Обозначается: z=x+iy. Символ i называется мнимой единицей. Обозначение:x=Rez; y=Imz.

Криволинейная трапеция - это фигура, ограниченная сверху графиком функции y=f(x) (f(x)≥0),
слева и справа соответственно прямыми x=a и x=b, снизу – отрезком [a;b] оси OX.

Математика – наука о количественных отношениях и пространственных формах действительного


мира

Математическое ожидание дискретной случайной величины Х - это число, приблизительно


равное среднему значению случайной величины, которое равно сумме произведение возможных

значений случайной величины Хn на соответствующие им вероятности pk: .

Механический смысл производной - это скорость изменения любого процесса. Например,


производная пути S=S(t) по времени t есть мгновенная скорость движения материальной точки, т.
е. V(t)=SI(t). Вторая производная пути по времени – ускорение, т. е.SII(t)=VI(t)=a(t).

Независимые испытания - это испытания (эксперименты), в которых вероятность появления


любого исхода в каждом испытании не зависит от результатов других испытаний.

Неопределенный интеграл функции f(x) - это совокупность всех первообразных для

функции f(x). Обозначение: , где знак называется интегралом,


функция f(x) – подынтегральной функцией, а f(x)dx – подынтегральным выражением.

Область определения функции y=f(x) - это множество тех значений аргумента x , при которых
функция y имеет смысл. Обозначение: D(f)

Область значений функции y=f(x) - это множество значений y, принимаемых функцией y=f(x)
для всех x из области определения D(f), т. е. при x D(f). Обозначение: E(f)

Правильной называется дробь, у которой модуль числителя меньше модуля знаменателя. Дробь,
не являющаяся правильной, называется неправильной, и представляет рациональное число, по
модулю большее или равное единице.

Первообразной функцией для функции y=f(x) на промежутке Х называется такая функция F(x),
если в каждой точке х на промежутке Х выполняется условие F'(x)=f(x)

Равные векторы - это cонаправленные коллинеарные векторы, имеющие равные длины.


Радиус – вектор точки - это вектор, соединяющий начало координат с произвольной
точкой пространства.

Сонаправленные векторы - это коллинеарные векторы, имеющие одно направление.

Сфера - это множество точек пространства, равноудаленных от данной точки О, называемой


центром, на данное расстояние, называемое радиусом.

Сложная функция - это функция, для которой область значений функции


содержится в области определения функции

Сочетания - это число комбинаций, состоящих из элементов, взятых из элементов, которые


отличаются хотя бы одним элементом. Обозначение и формула для подсчета числа

сочетаний:

Случайное событие - это событие, наступление или не наступление которого в некотором


испытании зависит от ряда случайных факторов.

Случайная величина - это переменная величина, которая принимает свои значения в


зависимости от исходов испытания.

Среднее квадратическое отклонение случайной величины х - это величина


где - дисперсия случайной величины х.

Точка максимума функции - это точка в окрестности, которой


функция определена и для всех точек этой окрестности, отличных от
выполняется неравенство:

Точка минимума функции - это точка в окрестности, которой


функция определена и для всех точек этой окрестности, отличных от
выполняется неравенство:

Теорема - это математическое утверждение, истинность которого устанавливается путем


доказательства.

Теория вероятностей - это раздел математики, изучающий закономерности, которым


подчиняются случайные явления и процессы.

Теорема сложения вероятностей двух событий - вероятность суммы двух событий А и В равна
сумме вероятностей этих событий без вероятности произведения этих событий:

Теорема умножения вероятностей двух событий - вероятность произведения двух событий


ровна произведению одного события на условную вероятность другого, вычисленную при
условии, что первое событие произошло:
Функция - это правило, которое каждому числу из некоторого множества ставит в
соответствие одно и только одно число у из множества . Обозначение: где -
независимая переменная, называемая аргументом ; -область определения функции; -область
значений функции.

Формула Ньютона-Лейбница - это формула для вычисления определенного интеграла от

непрерывной на отрезке функции f(x), имеющей первообразную F(x):

Формула полной вероятности -это формула для нахождения вероятности события А, которое
может произойти только с одним из n попарно несовместных событий образующих

полную группу:

Хорда - греч. слово horde – «струна», «тетива». Отрезок, соединяющий две точки окружности.

Число — основное понятие математики, используемое для количественной характеристики,


сравнения, нумерации объектов и их частей.

Число – абстрактная сущность, используемая для описания количества.

Целые числа — расширение множества натуральных чисел , получаемое добавлением к


нуля и отрицательных чисел вида .

Число е - это иррациональное число 2,7…, служащее основанием натурального логарифма

Экстремум функции - это локальный максимум и локальный минимум функции.

Экспонента (экспоненциальная функция) - это показательная функция .

Вам также может понравиться